Вы видите копию треда, сохраненную 16 марта 2018 года.
Скачать тред: только с превью, с превью и прикрепленными файлами.
Второй вариант может долго скачиваться. Файлы будут только в живых или недавно утонувших тредах. Подробнее
Если вам полезен архив М.Двача, пожертвуйте на оплату сервера.
Из самого себя - это элементарная частица. Бесконечно малого - это точечная частица.
Вообще-то она на твоей мамке стоит
Благодать - неземное проявление святого духа, что нисходя наполняет карманы батюшки благами.
Земля - формы сладкого хлеба. А стоит она на трёх слонах потому, что они зелёные.
Но если он бесконечно малого размера, значит меньше планковской длины? Тогда для освещения фотонами какого-либо объекта, потребовалось бы бесконечно большое время?
>что конкретно в солнечном свете является тепловой энергией? ИК диапазон?
да нет, по сути любое излучение превращается в тепловое в разной степени. Что-то черное будет поглощать и перерабатывать в тепло весь спектр, для которого это что-то черное.
Нет, ведь взаимодействие фотонов с электронами никак не связано с размерами фотонов, просто происходит процесс передачи энергии с поглощением фотона.
Погодите, частица - некий объект локализованный в пространстве, а теперь у него и размера нет, и не влияет ни на что, это минимум непонятно. Если фотон поглотился в одном месте, значит перед этим он излучился в другом месте, вот мне и интересно, в каком виде вылетает откуда-то фотон и какого размера.
Нет, это просто параметр, в том числе определяющий его энергию.
>>25145
Нисколько.
>>25147
Именно, понятие размера применимо только к составным частицам, у элементарных же его можно определять разве что как зону взаимодействия, которая различная для разных взаимодействий, плюс размазана в пространстве в виде вероятностного поля.
>>25148
>в каком виде
В виде фотона.
>какого размера
Никакого.
К биологам. С детства не могу понять, почему воду из родников безопасно пить, по крайней мере вероятность отравиться ей меньше чем водой из реки и т. п. В деревнях до сих пор ведь набирают и пьют, даже не кипятят. В земле же тоже бактерии живут, почему вода чаще чистая, чем опасная для здоровья ?
Ну ок, вылетает фотон, безразмерный, откуда он знает куда ему надо прилететь, и как он попадает в другую частицу?
>Погодите, частица - некий объект локализованный в пространстве
Неверно. Иди читай учебник или не делай покровосрывающих заявлений.
Из определения ясен же геометрический смысл. Собственный вектор - такой, на который оператор действует просто как умножение на скаляр из поля. То есть в этом "направлении" оператор действует как "сжатие" /"растяжение". Собственное число при этом показывает, во сколько раз он "растянулся".
В том и дело, что частицы не локализованы в пространстве. Квантовая механика всегда выделяет слово всюду жирным
В базисе собственных векторов, матрица становится диагональной. Это прекрасно. У тебя в этом базисе, подобное линейное преобразование, становится просто умножением одного вектора на другой.
Ок, а сможет ли параболическое зеркало в пасмурный день собрать хоть какую-нибудь энергию, учитывая что свет будет не прямой а рассеянный?
я хз, что туда вбивать, урл на статью получить не могу. да я криворукий мудак, который никогда не пользовался сайхабом
Там поиск есть. Еще можешь сначала найти в google scholar страничку. И открыть через Sci-hub
И еще вопросик - конечна ли таблица менбелеева? Слышал что новые элементы как то делают но на оч малое время - есть толк то от них? Или так чисто посмотрели и хуй сним.
>больше всего кислорода а потом только водород
в воде кислорода в 8 раз больше чем водорода, если сравнивать по массе
>Слышал что новые элементы как то делают но на оч малое время - есть толк то от них?
они ищут островок стабильности, где будут долгоживущие элементы с большим количеством нуклонов
По количеству, больше водорода. По массе кислорода. Это как, кого больше. Две кружки или ведро воды?
А вот про вселенную бубнят там того сего столько то процентов это массы или количества. И про землю - там тоже проценты и они отличаются от вселенских.
Кому не впадлу. Напишите как они эти проценты нашли. Ну и ваще за эту тему. И про таблицу менделеева- бесконечны ли элементы вней или есть конец - типо какаято постоянная таблицы менделеева за которой хуй гроб и вроде бы кладбище . ну вы уловили мысль. Че учоные думают? Типо после 500 элементов наша вселенная свернется в трубочку а вот если в ней есть 702 то это матрица. Надеюсь направление своих мыслей объяснил.
Он вылетает в определённом направлении и далее летит по прямой, пока с чем-нибудь не "столкнётся". Столкновение носит вероятностный характер, на большом расстоянии от электрона, например, есть маленький шанс поглощения фотона, на близком расстоянии шанс больше. Так или иначе происходит акт поглощения - это элементарное физическое событие, просто фотон исчезает, а электрон получает его энергию и импульс.
> Сколько сперматозоидов может попасть в яйцеклетку ?
Один, у неё есть специальный механизм, который сразу окукливает клетку после попадания первого и не пускает других.
> Сколько новых особей может развиться при попадании 1 сперматозоида(близнецы из 1 или 2)
В теории сколько угодно, на практике у человека доживают не больше четырёх. Даже если второй сперматозоид как-то сможет попасть внутрь, то ничего хорошего из этого не выйдет, зигота просто умрёт. Однояйцевые близнецы образуются уже при делении зиготы, бластомеры могут потерять связь друг с другом и тогда из каждого развивается отдельный организм, но с одинаковым геномом.
> Зависят ли ответы на эти вопросы от вида организма ?
Количество близнецов, которое может родиться - зависит, остальное нет.
> Есть ли понятие разнояйцевые близнецы или не одно яйцевые ?
Конечно, тебя в гугле что ли забанили? Они образуются, когда два сперматозида попадают в две разные яйцеклетки.
Нет ограничения, можно хоть миллион нуклонов в ядро запихать, да нейтронная звезда например, её можно считать за одно гигантское ядро.
Ответ на первый вопрос слишком сложный, ты не поймёшь.
Облака попускают инфракрасное излучение.
По твоей логике со звездой- где то далеко в таблеце должна быть эта самая звезда. Я к тому и клоню что эта таблица может дать гороздо больше того, чем мы пользуемся сейчас. И опять же в случае, если звезды в табличке нет. Значит есть какоето принципиальное ограничение. Какаято формула закон итп. Че великие умы современности думают. По этому вопросу?
Часть детей будут сильнее. Часть слабее. Часть будут обычными. Слабые поумирают. Останутся сильные и обычные. Если взять среднее от оставшегося (сильные + обычные), то это среднее будет сильнее чем обычно.
А вообще это зависит от того, какого нигера брать. Если афроамериканца, то он и без белой сучки будет здоровее. Потому что его предки-рабы подверглись очень жосткому селекционному отбору.
А если взять африканца, то результат будет ничем не лучше, чем если бы мы взяли европейца.
Проблема в том, что я никак не могу получить ссылку на саму статью, только на её описание. А вообще статья может нехилый тут срачь поднять среди мамкиных физиков.
Великие умы современности не обнаружили никаких формул и ограничений на это дело.
лол, ты думаешь, что кто-то подписан на эти вестники мухасрансков?
А где они его добудут? Его ж никто не производит в таких количествах, как этиловый.
>К биологам. С детства не могу понять, почему воду из родников безопасно пить, по крайней мере вероятность отравиться ей меньше чем водой из реки и т. п. В деревнях до сих пор ведь набирают и пьют, даже не кипятят. В земле же тоже бактерии живут, почему вода чаще чистая, чем опасная для здоровья ?
Бамп вопросу.
>>25153
>>25159
Не сосем то, что я хотел, но спасибо.
По КЭД он вылетает во всех направлениях, и каждый атом на пути взаимодействует с ним. И потом, если ты начинаешь "щупать" эти атомы, выясняется, мол "ой" один оказался возбужденным и скушал фотон.
Реально, но щас происходит не только антропогенное
>изопропиловый спирт
"Клиническая токсикология детей и подростков / Под ред. И.В. Марковой, В.В. Афанасьева, Э.К. Цибулькина: В 2-х томах.– СПб.: Интермедика, Специальная литература, 1999.– Т. 2.– 399 с."
Смертельной дозой (LD100) ИП при приеме внутрь для взрослых считают 240 мл, уровни смертельных концентраций варьируют от 0,04 мг/л у детей до 4,4 мг/л у взрослых. Метаболитом ИП является ацетон, эндогенный уровень которого у здоровых людей составляет 0,01 мг%. У больных диабетом 1-го типа, а также у голодающих людей ацетонемия может возрастать до 700 мг/л (Baselt R., 1982), причем в крови таких больных часто определяется и ИП, образованный эндогенным путем (Bailey D., 1990).
>К биологам. С детства не могу понять, почему воду из родников безопасно пить
Кишечная палочка, вибрионы холеры и прочие патогены живут только в гумусе (почва, навоз, фекалии). Плюс для их размножения требуется температура от 20 градусов. Родник вытекает из-под почвенного слоя и холодный.
>Он вылетает в определённом направлении и далее летит по прямой, пока с чем-нибудь не "столкнётся"
Значит можно проследить его траекторию и предсказать куда конкретно попадет? В том числе и в опыте с двумя щелями? И не совсем понятно про излучение фотонов, если они во все стороны поштучно от источника летят, тогда с какой скоростью выплевываются, чтобы попасть во все окружающие объекты, или они одновременно кучей вылетают?
Но я не исключаю, что мог и хуйню спиздануть. Я умею.
Гуглишь double Atwood machine
...
Profit!
Решение https://commons.wikimedia.org/wiki/User:Phys1csf4n
Пояснение https://physics.stackexchange.com/questions/118993/physics-double-atwood-machine
Еще одно http://physicstasks.eu/508/a-pulley-system
Человеческое решение через Лагранжеву механику а не ебучие силы https://www.colorado.edu/physics/phys3210/phys3210_fa15/lecnotes.2015-09-04.More_Examples_in_Lagrangian_Mechanics.html
>>25271
>проследить его траекторию
Понятие траектории в квантовой механике бессмысленно (за исключением бомовской теории)
>И не совсем понятно про излучение фотонов, если они во все стороны поштучно от источника летят, тогда с какой скоростью выплевываются, чтобы попасть во все окружающие объекты, или они одновременно кучей вылетают?
Мне кажется, тот анон имел ввиду континуальный интеграл, который можно проинтерпретировать как "частица двигается по всем возможным траекториям одновременно", но это скорее математическое описание, чем реально происходящее событие. Пусть меня анон поправит. Если тебя это интересует, возьми полистай Фейнмана/Хиббса, а там глядишь и до КТП дойдешь через пару лет.
Бамп вопросу.
>Смертельной дозой (LD100) ИП при приеме внутрь для взрослых считают 240 мл
Ну норм, если два с половиной литра чистого эталона залпом выпить, то тоже коньки отбросишь.
>Значит можно проследить его траекторию
Можно, но если ты это делаешь, то его квантовые свойства идут по пизде, интерференция в двухщелевом опыте пропадает.
>и предсказать куда конкретно попадет?
А это нельзя, если в процессе его полёта происходят отражения, потому что отражаться он может в совершенно рандомных непредсказуемых направлениях.
>И не совсем понятно про излучение фотонов, если они во все стороны поштучно от источника летят, тогда с какой скоростью выплевываются, чтобы попасть во все окружающие объекты, или они одновременно кучей вылетают?
Даже от светящейся точки под кнопкой питания монитора вылетают триллионы фотонов ежесекундно.
А что насчет преобразования материи в энергию? Каким образом это происходит на самом низком уровне? Что происходит с этими частицами? Они просто исчезают, высвобождая какой-то заряд?
Я тупой, поэтому объясните как-нибудь попроще.
Бамп.
Ты не можешь говорить про "размеры" элементарных частиц. Они "размазаны" в пространстве.
Матешу подтяни. В пересчете на 40% объемных это как раз пол-литра выйдет, чтобы коньки отбросить. Спроси у папы: спирт "Рояль" - он вкусный был? В нем было до 10% объемных изопропанола, так как делали его из опилок. Травилось очень много этим спиртом.
LD100 - это гарантированная 100% смерть. Для этанола - 250-400 мл, в некоторых источниках - 1200 мл (по-моему - речь о водке)
Вот еще отсюда http://mzdrav.rk.gov.ru/file/Toksicheskoe_dejstvie_alkogolja_18042014_Klinicheskie_rekomendacii.pdf :
>После приема внутрь даже 20 г 2-пропанола через 30-60 мин развиваются ранние симптомы отравления: боль в желудке, тошнота, рвота, понос, состояние тревоги или сонливости, могут быть судороги, запах ацетона изо рта, тахикардия, угнетение сухожильных
рефлексов, нарушения речи, атаксия, расширение зрачков, нистагм, энцефалопатия, потеря сознания, кома, повышение температуры тела, нарушение функции почек, анемия.
>Для этанола - 250-400 мл, в некоторых источниках - 1200 мл (по-моему - речь о водке)
Херня же полная, многие и по два литра водки выпивают на похуй.
>гарантированная 100% смерть. Для этанола - 250-400 мл
Так у нас по улицам живые мертвецы постоянно ходят, пошатываясь, I see.
грабли купи
>гарантированная 100% смерть. Для этанола - 250-400 мл
Самофикс: в Британской энциклопедии сноска - * кроме русских и поляков.
Неплохо получается: размера нет, то ли безразмерная точка, то ли где-то "размазан", траектории движения оказывается в принципе нет... А с чего тогда взяли, что фотон вообще существует?
>Человеческое решение через Лагранжеву механику а не ебучие силы
однохуйственно же, зачем школьника пугаешь. Пусть сразу записывает силы.
Какая милота, даже в саентаче пятнашки работают.
Как это? Взять, например, парафин, расплавить и испарить? Или наоборот, смесь углеводородных газов, сконденсировать в жидкость, затем - в парафин?
Чтоб перекаты не пилить - сделайте этот тред прикреплённым и бесконечным, как в /сс
https://2ch.hk/cc/res/229275.html (М) (там значки увидите).
Только картинку годную запостите и шапку.
https://youtu.be/A8XXYRvakao
> А с чего тогда взяли, что фотон вообще существует?
Ну наверное с того, что ты видишь что-то, кроме черноты?
1) Разлетелась бы к хуям при разгоне.
2) Если заклеить её скотчем и крутить, масса вырастет и выйдет черная дыра.
Понимаешь, тут сразу же видно, что ты не знаешь классической физики, и это основная проблема обмазывания научпоп квантмехом.
То, что ты смущен словом "размазан" однозначно говорит о твоём незнании классической физики волн.
Записаться в библиотеку и там читать фундаментальные труды по интересующим дисциплинам?
Так ведь книги частично уже изжили себя, сейчас есть интерактивные курсы и видеолекции это нагляднее и лучше воспринимается, чем книженция на полторы тысячи страниц.
Где общаться с умными людьми помимо двача?
Physicsforums, stackexchange, reddit
Составь план на год-два по интересующим тебя темам
Лично я не переношу видео, есть много отличных англоязычных книг для "начинающих"
Главное - тайм-менеджмент и периодичность, есть шанс взяться с ультра рвением и бросить всё через неделю
Так а что касается книг?
Хочу с интересными хипстерками продвинутыми пообщаться, с короткостриженными тяночками.. Они на всяких форумах водятся?
Пытаются ли сделать организмы, в строении которых есть незаюзанные природой 40+ аминокислот? Что из этого выходит?
Ну стоит ТП 10/0,4кВ транс 250кВА звезда-звезда. Три фазы, из центра нейтраль. С шины НН 2 отходящие линии с АВ по 160А.
Приходит в разетку фаза и ноль. Нахуй он нужен?
Симуляция C. elegans вроде тоже еду ищет. Хотя там слишком простая нейросеть, чтобы назвать это "поиском"
https://github.com/Flowx08/Celegans-simulation
Для того, чтобы искать еду необходима инициализация поисковых процессов в нейросетях,
активируемая подачей определённых нервных импульсов на входы нейронных сетей,
с учётом того, что эти импульсы самоорганизуются в организме при вызываемом голодом дисбалансе веществ в организме.
Восприятие состоит из чувственных данных (ощущений), и именно ЧУВСТВО голода искажает его, а не сам голод.
>Восприятие состоит из чувственных данных (ощущений), и именно ЧУВСТВО голода искажает его, а не сам голод.
Харчую. Всё в говне, а так много людей с ЧУВСТВОМ перманентной победы по улицам ходит, что поневоле задумаешься о величии разума над материей.
Разумеется, ты конченый мудак и несёшь хуйню, но главное, что ты ЧУВСТВУЕШЬ себя правым.
Мочератор! Опять /po/раша протекла!
Нихуй не пойму, как такое может произойти, объясните пожалуйста.
Лол.
У меня последняя пересдача через неделю, от меня требуют 190 страниц теории, которая мне нахуй не сдалась (решать матан я могу, это мне пригодится по моей специальности) Но нахуй мне будучи научным сотрудником нии доказывать что существует поверхностный интеграл?
Ладно, хотел узнать, есть ли какие-то действительно эффективные способы заучивания (зазубривания) матана? Сколько не учу, сразу из головы это вылетает
зазубривать не надо. Надо научится писать путь от утверждения А к Б: A \Rightarrow B
Понятия не имею, каких он целей планировал добиться, но это просто стыдно, для ученого, лить такую жижу.
Не читайте Ананасы.
>Но нахуй мне будучи научным сотрудником нии доказывать что существует поверхностный интеграл?
Это супернаивный вопрос, все его задавали сами и слышали (кто преподаёт) ещё в школе.
Школа, как по большей части и университет, учит учиться. Если ты идёшь в нии, то по крайней мере до этого должен был дойти сам.
Зазубривают те, кто не может или ленится понять. От тебя требуется систематично, с высоты птичьего полёта, каждое доказательство разбить на подразделы, тогда всё будет проще. Самое лёгкое - за деревьями не увидеть леса, потому что в доказательствах студенческого матана очень много техничных мелких деталей. Для этого нужно каждое доказательство сжать до нескольких предложений - представь, что пытаешься объяснить доказательство своей толстой мамаше.
После выхода в космос пророчили ИИ, путешествия во времени, космические путешествия и всё к этому шло, но вдруг шиш?
после ссср мировая наука встала, очевидно же
> Почему после 80х наука остановилась?
Ебать ты долбоёб, братишка. С чего ты взял, что остановилась?
> ИИ, путешествия во времени, космические путешествия
Такие сложные технологии зависят от кучи смежных. Например, нейросети относительно недавно стали широко использоваться, потому что появилось достаточно мощное железо.
Учить матан. Это как воду в решете носить. Люблю на экзах собсвенные доказательства приносить
Нет. Температура это статистическая величина. Ты не можешь сказать "молекула такой температуры"
>Нет. Температура это статистическая величина. Ты не можешь сказать "молекула такой температуры"
Я имел в виду не сами молекулы по отдельности, а общее количество молекул газа 1 и газа 2, если эти газы рассматривать отдельно от смеси.
Нет. Они будут одинаковой температуры. В этом и суть
Ты просто накидал терминов. Я говорю что для того чтобы искать еду, нужно чувствовать голод органами чувств. Которых у робота нет, в отличии от человека. Именно поэтому человек и обладает сознанием, это непрерывный параллельный процесс физического и психического.
Просто ты мамкин технарь редукционист.
Предел текучести при температуре 25 0С у чистого железа 50 МПа. Если чистое железо засунуть в атмосферу давление которого 50 МПа и охладить до 25 0С, то оно(чистое железо) изменит свойства? Станет мягким как пластилин, или текучей жидкостью? А что будет с чистым железом если засунуть в атмосферу давление которого равно пределу разрыва? Станет атомарным газом?
Повтори свой рисунок, только сначала сняв и уронив на корпус плюсовую клемму. Пизданёт.
Да я сам уже запутался. Теория гласит:
> необходимо обязательно снимать с аккумулятора сперва минусовую клемму, а потом плюсовую? Тип если плюсовая попадет на корпус, при одетой минусовой, произойдет короткое замыкание
Нихуя не происходит, в любом случае.
Если в электрике автомобиля есть какие-то конденсаторы - они могут дать резкий разряд.
Вообще, ИМХО, самый большой риск - то, что клемму мы снимаем руками. Если в этот момент человек случайно коснётся корпуса при неснятой отрицательной - он замкнёт собой цепь, и его ёбнет током. Если сначала снимать отрицательную - этого не будет.
всмысле? Ну вот возьму я диск, скажем, болгарки. Он весит допустим 200 грамм. Выходит если я его раскручу он будет весить килограмм?
Ок, пусть даже имеется ввиду инертная масса - если я сдвину включенную болгарку нужно будет приложить усилие, равное сдвигу кирпича?
Есть инфа, что если взяться двумя руками за клеммы (электроды) аккумулятора, то не произойдет нихуя, тк сопротивление слишком высоко.
От инертной массы отказались. Ибо при больших скоростях теряется её универсальность, и тело "сопротивляется" в одну сторону сильнее чем в перпендекулярную.
Да, гравитационная масса будет расти, и если болгарка невероятно крепкая то вырастет и станет черной дырой.
да погоди ты с черной дырой. Допустим раскручивает ее не до сверхсветовой: она больше от этого притягивается к Земле?
лол, но ведь это бред. Сколько не раскручивай тело относительно своей оси, на весах будет постоянное значение.
учу биохимию, и вот проблемка. я никак не могу вникнуть в систему метаболизма у микроорганизмов
ну вызубрить то это без проблем, но общая картинка не складывается
анон, смари, в начале же идет, допустим, гликолиз, ну или кдфг, или пф, плевать
вот пируват
потом окислительное декарбоксилирование в ацетил-коа
потом цикл кребса
а потом окислительное фосфорилирование?
в общем, не обращайте внимания на эту бессвязную речь, но если кто-нибудь из вас сможет выдать мне дельный лучик света, буду пиздец как рад
Ты просто медленно раскручиваешь. Масса почти не меняется. Но при скоростях сравнимых со скоростью света начинается эта херня.
Потому что всё оказалось не так просто, как казалось поначалу, и люди приблизились к пределу сложности, которую могут воспринять их мозги, но эта проблема скоро решится с изобретением ИИ, либо с генными модификациями человека.
> всмысле? Ну вот возьму я диск, скажем, болгарки. Он весит допустим 200 грамм. Выходит если я его раскручу он будет весить килограмм?
Только не килограмм, а 200.0000000000000001 грамм, а вот если ты раскрутишь до субсвета, то может и на килограмм утяжелиться.
>анон, смари, в начале же идет, допустим, гликолиз, ну или кдфг, или пф, плевать
>вот пируват
>потом окислительное декарбоксилирование в ацетил-коа
>потом цикл кребса
>а потом окислительное фосфорилирование?
Да, всё так, в результате цикла кребса получается НАДН, который используется в окислительном фосфорилировании.
Что-то типа беговой дорожки подойдёт?
>Как мне из обычной живородящей ящерицы вывести бипедальную? Какие мануалы курить?
Вот этих няш выращивай в таких условиях, чтобы им почаще приходилось бегать.
анекдот_про_женскую_баню.тхт
а есть какойпнибудь типичный процесс после окислительного фосфорилирования или его можно назвать концом?
Что значит "крутиться"? Вращаться вокруг солнца или своей оси? Если первое, то её на каком-то этапе выплюнет в космос.
Если вокруг оси, то её распидорасит, когда сила ньютона (центробежная сила) превысит силу сдерживающую вместе молекулы.
Вообще это всё легко гуглится, все расписано доступным языком. И еще погугли, что такое "вес", и что такое "масса", это совеншенно разные понятия СПОЙЛЕР: НИСМОТРЕТЬ: масса это физическая величина, а вес это ебать сила, и измеряется в ньютонах.
И проверяй всё, что тебе говорят на двачах.
После него получается АТФ, который используется во ВСЕХ процессах, где нужна энергия.
Но они ведь откладывают яйца! Тогда, как вывести живородящих василисков?
Листал прошлый тред. Держи - Rondo Duo, это новелла, но на порносайтах есть вырезки со всеми порносценками. Их дохрена и они довольно специфичные.
Хочу купить автоклав, это такая штука для консервирования.
Суть в том, что в ней вода нагрета до 120 градусов и давление 5 атмосфер.
Вот что случится если крышку вдруг сорвёт? Ну мало ли, ушко так отломится из-за косячной сварки.
Что будет?
Вся вода моментально превратится в пар, заполнит все комнаты в квартире, выбьет все окна и сварит заживо всех находящихся в ней?
Или при расширении всё быстро охладится и ничего опасного не случится и можно сидеть рядом с автоклавом пол часа и следить за ним?
> Или при расширении всё быстро охладится и ничего опасного не случится и можно сидеть рядом с автоклавом пол часа и следить за ним?
This, даже не вся вода испарится, а лишь небольшая часть, потому что на испарение уходит во много раз больше энергии, чем на нагрев.
>потому что на испарение уходит во много раз больше энергии, чем на нагрев.
зато при конденсации перегретого пара на чье-то ебало выделяется так же много тепловой энергии.
ДОШЛО, спасибо, анон
>>25695 (Del)
Клапан там есть, но это полу-самодельный какой-то газовый баллон с приваренной крышкой.
Мне интересно что будет с водой нагретой до 120 градусов при 5 атмосферах если эту крышку оторвёт.
Если вся она моментально превратится в пар, я лучше куплю какой-то электрический автоклав и на балкон поставлю, пусть там взрывается сколько хочет.
Но он дорогой пиздец.
А если ничего не будет, то и не страшно.
Ну смотри, мамка же твоя тебя как-то сделала. А ты говоришь невозможно.
https://www.youtube.com/watch?v=TTaEOpNgQIc
https://www.youtube.com/watch?v=5HlEmEaa2Hw
Савельев говорит, что оружие на биоматериалах собрать как нехуй делать можно, Марков отнекивается.
пусть сделает, блять.
этого савельева уже на федеральные каналы зовут.
Профессор из мгу все правильно сказал, и еще пример приводил, как я помню: про комаров и т.д.
Не верь никому, покупай ОЗК и ребризер.
И пройдите опрос, кому не сложно, для анализа
https://docs.google.com/forms/d/e/1FAIpQLScDYMk9ys0wTGfRqUreFL1SY ... g/viewform
Ну не траль.
>>25734
Расскажите пожалуйста подробнее про
>(замедляя вращение Земли)
Я всегда думал что ветер возникает из-заразности в давлении(температуры), приливы и отливы из-за вращения луны. Причём тут вращение земли ? Почему нельзя именно отнимать энергию у ветра и воды, чтобы земля не замедлялась ? Такое замедление может привести к катастрофе или к тому что в сутках станет 25 часов ?
Всё гораздо сложнее. Можешь отсюда начать.
https://ru.m.wikipedia.org/wiki/Приливное_ускорение
Если не хочешь по-хардкору, то советую маленькую книжку из библиотечки Квант, там "Земля" в названии, не помню точно.
хз, говорит вполне годные вещи, но что-то все же тут не так. Слишком любительские рассуждения которые без массовой проверки на МРТ выглядят нелепо.
Если приложанная сверху сила увеличивает вес, то что из этого следует?
Почему мягко говоря? Мягко и вредна, ты от одного стакана не умрёшь.
Посоны, вот если за 10000 световых лет от земли ебанёт какая нибудь сверхмассивная звезда, то будет катаклизм вокруг взрыва, но последствия от места пиздеца дойдут до земли не раньше чем свет, почему так?
Разве это не означает что сам космос, само пустое пространство не совсем пустое, если оно ТОРМОЗИТ распространение того же взрыва звезды?
То есть что если пустое пространство не пустое, оно как эфир чтоли, в котором есть свои свойства, типа ограничение на скорость света того же?
А, забыл, почему в макро и микро мире объекты стремятся стать шарообразными?
Вот например, если я в стакан воды капну каплю масла - она станет шаром в воде из-за среды, в космосе, в "пустом пространстве" тоже самое, что звезды, что планеты - шары, да что там планеты, АТОМЫ, мельчайшие частицы которые заполняют вселенную - шарообразные.
Мальчик, ты дурак... ой да мы же в треде дураков. Просто свет сам по себе летит с той скоростью, с которой он летит. С другой стороны, да, пространство не есть совсем НИЧЕГО, у него есть св-ва, например, скорость света в нем или поле Хиггса, придающее массу лептонам...
>>25823
Шарообразная форма имеет минимальное отношение площади к объему. В случае капли воды, например, это минимальная потенциальная энергия поверхности (из-за силы натяжения).
>АТОМЫ шарообразные
ой не пизди, на картинку посмотри
Вопросами гравитации занимается ОТО, а ты пока только с СТО ковыряешься. Читай дальше...
Котаны, обладает ли этиловый спирт слабой щелочной реакцией? И вообще почему крепкий алкоголь водочку гораздо легче запивать чем-нибудь кислым? Или традиционно коньяк с лимоном. Так гораздо меньше жжет. Почему так?
>дойдут до земли не раньше чем свет, почему так?
потому, что свет, как электромагнитное взаимодействие, двигается с фиксированной скоростью, а материя, имеющая массу покоя, двигается всегда медленнее. Значит сначала прилетит свет, потом все остальное говно.
> если пустое пространство не пустое
а оно и не пустое
>в котором есть свои свойства
а они есть
Нет. Если проглотить просто спиртягу, то давиться будешь гораздо больше, чем от лимона. А если с лимоном или даже просто запить водой с лимонной кислотой, то всё быстро проходит.
Гугли QED, вторичное квантование, вот это вот всё, Если кратко, то КЭД - простейшая калибровочная теория с симметрией U(1), и электромагнитное поле выступает в роли калибровочного поля. Ебошишь диаграммы Фейнмана, считаешь матрицу рассеяния. Феноменологическое описание рискует скатиться в научпоп про реально существующие виртуальные фотоны, поэтому лучше без понимания математики этим не баловаться.
Советую и тебе и мимокрокам отличную (и доступную) книгу "КЭД - странная теория вещества и света".
Автор пишет: "выражение в скобках будет мало отличаться от единицы". НО ПАЧИМУ?! Ведь это выражение должно равняться нулю, чтобы при умножении на x^3 дать итоговый ноль, т.к. сам x не может быть нулем по условию. Следовательно, сумма слагаемых в скобках правее единицы должна быть -1.
Что это: ошибка в тексте или моя тупость?
Бля, если раскрыть скобки, то получится первое уравнение. Значит, ошибки в тексте нет. Ну пиздец...
вот после таких ответов и плодятся катющики у нас
Меняется ли количество энергии, которую тратит мой организм на эти десять раз в первый день тренировки и в последний? Как это изменение связано (и связано ли?) с субъективным ощущением облегчения жима?
- чем такой особенный углерод и могут ли другие элементы объединяться в такие же длинные цепочки?
- является ли кристаллическая решетка одной большой молекулой, или же она состоит из отдельных, с большим(относительно) расстоянием между ними?
>чем такой особенный углерод и могут ли другие элементы объединяться в такие же длинные цепочки?
Ты сам ответил. На самом деле там прочность связи приличная. и в то же время двойные и тройные на уровне.
> является ли кристаллическая решетка одной большой молекулой
зависит от определения молекулы. Но это немного размытые понятия. Когда начинаешь изучать, забиваешь хер обычно. определения на суть вещей не влияют.
>Ты сам ответил.
чем такой особенный углерод что он может образовывать длинные цепочки а другие нет?
>На самом деле там прочность связи приличная
от чего это зависит? Есть ли еще подобные элементы?
>зависит от определения молекулы.
ну, насколько я понял, связи между атомами внутри молеклы - химические, и чтоб их разорвать механически нужно сильно постараться. Тем не менее, кристаллическую решетку разломать довольно легко.
Ну разломай алмаз. Там вполне себе большая молекула.
>от чего это зависит? Есть ли еще подобные элементы?
Если я скажу от квантовой химии. Тебе станет понятнее?
Блять, дает ли НАУКА повод усомниться в правильности теории эволюции? Какие сейчас есть НАУЧНЫЕ доводы против теории эволюции?
Да, конечно дает. Иначе это была бы не наука. На вики даже написано не слишком мало. https://ru.wikipedia.org/wiki/Синтетическая_теория_эволюции#Критика_синтетической_теории_эволюции
Ок, наука продолажет развиваться. Ты не открыл мне Америку. Кроме креационизма есть ли альтернативы теории эволюции?
Что именно ты понимаешь под теорией эволюции? Теорию эволюции в смысле биологии? Тогда какую именно? Если классический дарвинизм, то по современным меркам он недостаточно научен. Если современное учение об эволюции, то оно весьма неоднородно и в его рамках существует несколько, порой весьма несовместимых друг с другом, концепций.
Если же попытаться выделить у них что-то общее, то получаем постепенное развитие жизни, сопровождаемое генетическими изменениями, адаптациями, возникновением и вымиранием видов в рамках изменяющихся экосистем. Эти тезисы - часть современной научной парадигмы. Можно ли в них сомневаться? Да на здоровье! Можно ли их опровергнуть? В общем случае нет. Доктринальные споры - моветон. Либо наука базируется на этих тезисах, либо в какой-то момент отметает их на фоне более масштабного процесса смены научной парадигмы. Взять ту же концепцию эфира. Кто-нибудь ее как следует по-научному опроверг? Нет. На него просто забили, т.к. взгляды на мир поменялись. Геоцентризм кто-нибудь опроверг? Тоже нет. Сначала его просто заменили на гелиоцентризм, а потом появился Эйнштейн со своей теорией относительности и сказал, что все однохуйственно и зависит от того, что считать началом системы отсчета. Точно также и с эволюцией.
Ну не знаю... Что может придти на место эволюции? Это по-моему непоколебимая истина? Почему развелось так много скептиков, в корне отрицающих современное научное толкование эволюции?
>>25878
>есть ли альтернативы теории эволюции?
Подумай сам. Вот мы смотрим на природу и видим: животные приспосабливаются к условиям окружающей среды; жрут друг друга; виды появляются и вымирают; в ДНК происходят изменения и от этого меняются и сами организмы; происходят миграции; появляются, пропадают и видоизменяются биомы... Люди за этим наблюдали достаточно долго, пока пришли к одному выводу: бля, так это же все взаимосвязано! Давайте так: развитие жизни в общем виде назовем эволюцией; объектами эволюции станем считать виды, популяции и экосистемы; наличие и доступность пищи, присутствие конкурирующих и находящихся выше по пищевой цепочке видов, условия окружающей среды и тому подобное назовем факторами эволюции (в частности, принято выделять целый комплекс факторов и связыннх с ними процессов, называемых естественным отбором). Ну как-то так. Можно ли придумать этому альтернативу? Да сколько угодно. Например, сказать что на самом деле это все не взаимосвязанно или же взаимосвязанно совсем не так, как сегодня принято считать. Вот мы и вышли из коцепции эволюционизма. Но чтобы двигаться дальше в этом направлении, нужен нихеровый такой бэкграунд эмпирических и экспериментальных данных, методология, философское обоснование, в конце концов. Но главное - это насущная потребность, общественный запрос, исходящий из научного сообщества. На данный момент - эволюционизм более-менее соответствует нашим предсталениям о мире, поэтому и менять его на что-либо смысла нет. Это не вопрос выбора между эволюцией и креационизмом. В конце концов, существует и теистический эволюционизм. Это просто вопрос прежде всего мировоззренческий. На данном историческом этапе мы воспринимаем мир таким образм, что нам удобно описывать его в терминах эфолюционного учения.
>непоколебимая истина
Да, кстати, вопрос об истине, вообще-то, скорее философский, а не научный. Уж слишком специфичен этот термин. В рамках науки мы обычно говорим об определенной аксиоматике и постулировании.
Для возникновения чорной дыры нужно чтобы вся масса системы сконцентрировалась внутри гравитационного радиуса. У тебя часть массы тела за пределами гравитационного радиуса и К1 за пределами гравитационного радиуса. Вот из-за этого твоя система не стала чорной дырой.
Энергии тратится примерно одинаково в обоих случаях (на самом деле больше тратится на накачанную мышцу, но разница мизерная)
Субъетивное ощущение облегчения связано с тем. что у мышциы увеличивается площадь сечения. Из-за этого уменьшается сила натяжения на единицу площади. Например, если гантеля весит 1 кг, площадь попереченого сечения мышцы в начале S1 = 5 см2, а площадь поперечного сечения в конце эксперимнта увеличена до S2=10 см2, то получается, что:
в начале эксперимента сила натяжения P1 = 0.2 кг/см2, а в конце эксперимента сила натяжения P2 = 0.1 кг/см2
>Геоцентризм кто-нибудь опроверг? Тоже нет.
Хуйня вопрос. Суть геоцентризма в том, что Солнце якобы двигается по такой орбите, центр которой расположен в внутри Земли. Если бы это было действительно так, то при такой скорости движения Солнце улетело бы в дальние ебеня из-за центробежной силы.
>чем такой особенный углерод и могут ли другие элементы объединяться в такие же длинные цепочки?
Особенность углерода в том, что его соединение с кислородом может растворяться в воде и в воздухе в больших количествах при земных давлении и температуре. Поэтому он годится для создания органических соединений в земных условиях.
В такие же длинные цепочки может объединяться и кремний.
>В такие же длинные цепочки может объединяться и кремний.
...аккуратнее с такими заявлениями. Или придется бежать за пруфами что "такие же". Кремний не может в кратные связи. Кремний не может в ароматичность. Кремний не имеет такие крепкие связи, и легко окисляется, да и энергию активации окисления имеет мизерную.
Признаю свой обосрамс. В цепочки может объединяться оксид кремния, а не чистый кремний.
А хотя нет, не обосрался. Есть же силаны - кремний-водородные цепочки. https://ru.wikipedia.org/wiki/Силаны SinH2n+2. Правда в земных условиях они не устойчивы.
Почему нагрузка способствует увеличению площади сечения? И как это происходит?
Какой механизм связи между субъективным ощущением в начальной и в конечной точках и изменением площади сечения?
>Почему нагрузка способствует увеличению площади сечения? И как это происходит?
Организм реагирует на стрессовые нагрузки (да, для увеличения мышечной массы нужны на столько сильные нагрузки, что они вызывают стресс) сложной биохимией, в которой принимает участие тестостерон. Подробности биохимии расписывать не стану, потому что ты пока не готов. В результате действия этой биохимии мышечные ткани накапливают гликоген - топливо для работы мышц. От гликогена мышцы распирает, увеличиваются их размеры.
>Какой механизм связи между субъективным ощущением в начальной и в конечной точках и изменением площади сечения?
Механизм в том, что на самом деле мы ощущаем не вес, а механическое напряжение мышечных волокон. Ощущаемое напряжение прямо пропорционально силе, с которой гантеля притягивается к земле и обратно пропорционально площади сечения мышцы. От сюда и получается формула P = m/S, где Р - ощущаемое напряжение мышцы, m - масса гантели, S - площадь сечения мышцы.
Аналогично работают весы типа "безмен" (пикрелейтед) - они показывают не сам вес, а растяжение пружины. Где растяжение пружины эквивалентно её напряжению. Ну а связь между напряжением и площадью сечения тебе уже пояснили математически.
Тогда мы получим три тела, размещённых в вершинах равнобедренного треугольника в идеальных условиях.
Попробуй переформулировать свой вопрос.
Уточняющий вопрос - как расположен твой треугольник в трёх-мерном пространстве относительно плоской горизонтальной твёрдой поверхности?
Может ли быть так, что в процессе эволюции произошло так, что человек может жить только на Земле, и его сознание воспринимает только ограниченную часть вселенной? Дорога в открытый космос нам закрыта и нечего думать о звёздах. В текущем состоянии, когда человек это скопление клеток это невозможно. Другие же "существа" могли эволюционировать до другого состояния. К примеру "существо" имеет размеры одной элементарной частицы. Это состояние элементарной частицы позволилобы им путешествовать по открытому космосу, свободно обменивать информацией на огромные расстояния. Я сейчас пишу в огромных условностях, но есть ли такая возможность и кто нибудь уже писал про такую наркоманию?
Я понял твой ответ, но не понял, что не так в моём вопросе:
>Какой механизм связи между субъективным ощущением в начальной и в конечной точках и изменением площади сечения?
Я связал субъективные ощущения в разные моменты, типа разница субъективно ощутима. В чём у меня ошибка и почему так делать нельзя, можешь пояснить?
Бля, ну ты понил, задача трех тел. В космосе, где нет вообще нихуя, помещаем три неподвижных тела, на равном расстоянии друг от друга. Как они поведут себя? Просто столкнутся в середине треугольника?
Бля, почему рисовать нельзя в ебучем ссаи?
Во первых я не понял. Ты говорил про равнобедренный треугольник, а у тебя на картинке не просто равнобедренный, а ещё и равносторонний.
Во вторых, если взять идеально одинаковые тела, расположить их на идеально равностороннем треугольнике идеально ровно и идеально неподвижно в идеально равномерном гравитационном поле и отпустить идеально одновременно, то они столкнутся в центральной точке.
Вопрос был о связи ощущений в разные моменты между собой и с изменением площади сечения. Ответ пояснил связь отдельно взятого ощущения с отдельно взятой площадью сечения.
Ошибка не ошибка, но несоответствие ответа вопросу.
>связи ощущений в разные моменты между собой
Связь в том, что ощущает один и тот же человек с одним и тем же мозгом.
Поясняю. Сила мышцы ограничена только её физической прочностью. Т.е. при подаче слишком мощного нервного импульса мышца может порваться. Соответственно, чем меньше площадь сечения мышцы, тем меньший груз нужен для её разрыва. Дрыщ, например, может надорваться и от 100 кг. А кочка может и 200 спокойно поднять.
Однако, у животных (и у человека тоже) в мозгу есть защитный рефлекс, который, при превышении некоторой нормы Pкрит посылает расслабляющий импульс, и таким образом не даёт мышце опасно перенапрячься. Ркрит - это постоянная величина.
Таким образом, если в начале эксперимента подобрать массы, для которой будет справедливо равенство mкрит= PкритSнач, а затем в течении эксперимента добиться увеличения площади сечения до Sконч, то получаем:
Sнач < Sконч
и в следствие этого получаем
mкрит < PкритSконч,
т.е. масса к концу эксперимента будет казаться более лёгкой, чем ощущалось в начале эксперимента.
Дохуя кто. Овер9000 результатов в Яндексе.
P.S.
Написанное справедливо для нормальных людей.
Однако есть ещё особая категория ненормальных людей. Называется "спортсмены". Спортсмены, помимо наращивания мышечной массы и ебли метаболизма тренируют ещё и нервную систему, приучая свой мозг перевозмогать защитные рефлексы. Соответственно у них Pкрит не постоянная величина, а возрастает в процессе тренировок.
Алсо есть ещё одна категория ненормальных людей. Называется "долбоёбы". Долбоёбы поднимают Pкрит с помощью энергетиков и прочих нейростимуляторов.
В обеих ненормальных категориях случается пикрелейтед.
Т.е. по факту и масса и напряжение к концу эксперимента перестают быть критическими, хоть ты так их и обозначил в последнем неравенстве?
>Почему эволюция не заебашила всем регенерацию оторванных органов, ведь это такой-то буст к выживаемости?
Потому что такая регенерация замедлила бы эволюцию этого вида, т.к. особь, по глупости потерявшая конечность, вместо того, чтобы умереть и изъять свой ущербный генотип из генофонда популяции, вместо этого выживет, наплодит таких же идиотов, которые будут отнимать ресурсы у более осторожных сородичей. Эволюционно более выгодно позволить этой неудачливой особи умереть и освободить жизненное пространство для молодняка.
Зависит от определения "живое"
>т.к. особь, по глупости потерявшая конечность, вместо того, чтобы умереть и изъять свой ущербный генотип из генофонда популяции, вместо этого выживет, наплодит таких же идиотов, которые будут отнимать ресурсы у более осторожных сородичей.
мамкин Гитлер пытается в эволюцию. Если особь выжила, значит она, блять нормальная. И закрепится тут должен наоборот ген регенерации а не наоборот. Цель эволюции - тупое выживание, а не создание сверхнации.
Двачаю
Нет, есть только нечёткие.
Следующими этапами будут: как сделать ее человеческого роста, как сделать млекопитающим с молочными железами четвертого размера.
Такая особъ даст меньше потомства.
Твоя "нормальная" будет тратить энергию на отращивание оторванной конечности вместо того, чтобы тратить эту же энергию на на производство яиц и сперматозоидов. Меньше яиц и сперматозоидов -- меньше потомства. Меньше - по сравнению с теми, кто не тратит энергию на регенерацию. Чем меньше потомства, тем медленнее эволюционное развитие. В итоге такой вид проиграет в конкурентной борьбе другим видам, которые эволюционируют в той же экологической нише. Особенно это касается тех видов, для которых давление естественного отбора направлено на развитие мозга.
Три человека: Рудольф Клаузиус, Людвиг Больцман и Джеймс Максвелл некоторое время считали смыслом своей жизни в развитии идей Ломоносова. Примерно так.
Спасибо,выручаешь.
Виртуальная жизнь.
Надо бы дополнить, что жизнь может эволюционировать. Компьютерные вирусы пока этого не могут.
2. Если взорвать термоядерку в неразрушимом прозрачном ящике:
-будет ли энергетический выход намного больше?
-насколько такая вспышка будет отличаться от взрыва? Будет ли грибок, ударная волна?
1. Зависит от того, какой грунт считать сухопутным. Вариантов сухопутного грунта хуева туча от сухих песков Сахары до влажной почвы арктической тундры.
2. Зависит от размера ящика. В ящике размером с бомбу и в ящике размером 100х100х100 километров эффекты будут разные.
>-будет ли энергетический выход намного больше?
Т.к. ящик прозрачный, то энергетический выход будет такой же.
Да, я это имел ввиду.
Ну вот мы сделаем два шага. У нас есть два исхода 0,0,2,2. Почему нужно брать их среднее квадратическое, а не среднее арифметическое?
существуют ли полимеры с температурой плавления/сублимации/разрушения более 500-800 градусов?
существуют ли твердые амальгамы(сплавы со ртутью), с некими особыми свойствами, используемые не как промежуточное звено в отделении металлов, а как собсно сплав(конструкционный, например)?
И вообще, есть ли примеры использования ртути в каких-то материалах, или еще где?
Так непонятно в книжке
Бамп
Смотря какие полимеры, есть неорганические полимеры с температурой плавления более 4000 градусов. Из органических вроде свыше 700 градусов нету.
Поговаривают что межзвездное пространство это не чистый вакуум а сильно разряженный газ. Возможно ли отталкиваться от него, скажем, ионным движителем, так же как самолет отталкивается от воздуха?
Диванно-теоретически наверно энерговыход будет больше. Но научно никто не проверял по причине невозможности такой проверки.
Лол нет. Тред был на столько скучным, что никто не сохранил.
Свойства изменятся. Железный предмет станет прочнее, твёрже.
Зависит от того, как прилагать давление. Если приложить давление неравномерно, т.е. с одной стороны давить, а с другой - не давить, то опытный образец деформируется. Например, так можно сделать с наглухо запаянным стальным баллоном, оставив внутри давление в 0.1 МПа (это 1 атм.) и поместив этот баллон в барокамеру с давлением 50 МПа (это 500 атм.).
А если давить со всех сторон одинаково, т.е. внутри и снаружи по 50 МПа, то баллон станет твёрже и не деформируется.
Объясните простыми словами без формул. Можно с графиками.
Без формул обяснение ентропии укладываеться в фразу "Уровень хаоса в системе" безпорядок в молекулярном смысле
больше по сылке (формулы залепи изолентой)
https://www.youtube.com/watch?v=M4pws5kp5Z0&list=PLxGo9dxQkqWDdGo6sCI4BarYeac_jbMiW&index=19
советую посмотреть весь сборник по термодинамике
Удачи анон
Когда мне говорят по хаос, мне мысленно представляются хаотично движущиеся во все стороны молекулы в газе. Это и есть энтропия? Если да, то я не понимаю, как эта хаотичность может быть большей или меньшей.
Если нет, то я не понял, что означает хаос.
Почему говорят, что люди, которые реже улыбаются - быстрее стареют и обрастают морщинами? Наоборот же, мышцы лица не изнашиваются и сохраняют форму. Так как?
В классической термодинамике, без статистики. Энтропия это просто аддитивная функция состояния. Как объем, или масса. Без размышлений о том, "что это". Сириусли. это потом уже прикрутили ответ.
Объём я могу пощупать и увидеть.
Массу могу приподнять.
С энтропией не понятно. Как она ощущается?
Блин, от ваших объяснений у меня такое ощущение, будто я слепой от рождения и мне пытаются объяснить, что такое красный цвет и чем он отличается от зелёного.
Энтропия родственная с температурой. Температуру тоже же ввели.
температура в термодинамике это частная производная внутренней энергии по энропии
(dU/dS)p,n
Температуру я могу пощупать пальцем.
Как пощупать энтропию?
Или ты хочешь сказать, что это просто какая-то ебанутая функция, у которой нет осязаемого физического представления, типа как... блин, я не знаю с чем сравнить.
Я тут недавно натолкнулся на видео, где механический кпд около 50%.
Такое вообще возможно?
http://www.teslatech.com.ua/index.php?option=com_content&view=article&id=68&Itemid=71
Насколько я знаю, в мощности до киловатта обычные паpовые двигатели куда лучше туpбин
Это лишь означает, что ты сам не понимаешь, что такое энтропия, а лишь тупо копипастишь чужие слова, как китайская комната. Нахуй ты вообще влез.
Пиздец... Внутренню энергию ты тоже пальцем не ощутишь. Но энтропию можно измерить косвенно. Скажем так. Энтропиеметра не существует.
Но ты блять в контексте читай. Я тебе про "классическую термодинамику". В ней энтропию запостулировали тупо как функцию. После, ребята умные пояснили что это такое. Но ты же не спрашивал у меня. что такое в статистической термодинамике энтропия.
Какой именно кпд, туpбинный или механический?
Да ты и на габаpиты внимание обpати, дpугие туpбины всосут на таких pазмеpах.
>У традиционных турбин кпд больше 95%. Нахуй нужна твоя 50% тесла.
Тут вопрос о масштабе, при мощности до киловатта никакая лопаточная турбина таких параметров ни за что не выдаст, если не ошибаюсь, то оно даже ДВС обходит, хотя тут весьма споpно.
Энтропией называют мерой хаоса, хотя по мне это мера полной одинаковости. Определения вряд ли могу дать, могу привести примеры для понимания.
Пример1. Когда в системе(путь будет запаяная колба) все частицы равномерно распределены по объему, на одинаковом расстоянии друг от друга, имеют одинаковую кинетическую энергию и если это смесь разных частиц(например CO2 и H2O) то распределении частиц тоже равномерное, т.е. в любой области объема соотношение частиц одинаковое как и во всем объеме системы(а ни как в реальном мире, где в самом куске мела концентрация примесей равна 2%, а возьмёшь разные маленькие области этого мела так там концентрация примесей скачет от 0% до 100%) в этом случае энтропия системы максимальна, т.е. отсутствуют разности потенциалов(разность температуры, разность давления, химический потенциал, электричесое напряжение и т.д.) - энергия как бы в системе есть, но что бы её оттуда достать нужно будет потратить дохуя работы, все частицы друг от друга не отличаются, кроме разности сорта(CO2 и H2O).
Пример2. Когда в системе частицы полностью заполнили только самую минимально возможную область объема системы, кинетическая энергия частиц скачет от 0 до бесконечности, концентрация примесей тоже скачет скачет от 0 до 100% в разных областях заполненного объёма системы - то энтропия системы минимальна. Т.е. когда всевозможные потенциалы имеют максимально возможное значение. В данном случае чтобы извлекать энергию из системы даже работу не надо затрачивать, она сама выделяется.
Т.е. когда говорят что энтропия -это мера хоаса, беспорядка. То под хаосом/беспорядком понимают меру одинаково распределенной энергии в системе. Энтропия максимальна - любая область системы ничем не отличается от всей системы в целом, все частиц одинаковы. Энтропия минимальна - любая область системы уникальна и не похожа на всю систему в целом, все частицы уникальны, нет не единой похожей.
>>26124
А вот хуй. В твоих примерах разности потенциалов как и прочие "отличия" в газе будут, как и локальные неоднородности концентрации. Если взять 2 любые точки(области, по объему стремящиеся к нулю но не абсолютные), то в них вполне себе будут различия, энергии будут нести в себе мало(из-за размера), но таких точек там практически столько же сколько и молекул, и в сумме энергии можно выкачать даже больше чем в примере 2.
Проблема в том что эти области слишком раздроблены, в таком случае энтропию можно назвать мерой раздробленности неоднородностей. Возможность выкачивать энергию из системы идет через весьма странное правило: чем меньше преобразователь энергии относительно локальных областей неоднородности, тем больше энергии(опять же, учитывая пропорции в размере) ты сможешь взять. Данное правило лично для меня не поддается объяснению, как и возможность существования демона Максвелла: если представить, скажем, море, и на нем большой(в сотни гектар) плот, состоящий из отдельных поплавков, каждый из которых относительно других связан механически так чтобы преобразовывать разность подъема в крутящий момент, затем все это передавая на некий единый вал, то такая система вроде как будет работать. Что мешает то же самое повторить на микроуровне - хз.
Но это чисто энергетическая сторона, чтобы понять что такое энтропия я думаю еще стоит копнуть в сторону информационной энтропии, потери информации, а затем охуеть оттого что информация таки материальна и несет в себе энергию. С этого момента энтропия - философский термин, а чтобы понять философию нужно долго самому раздумывать над всей этой хуйней.
в газе будут, как и локальные неоднородности концентрации.
В классике да. В квантоваом случае на бесконечном времени всё спокойно. вроде
>В квантоваом случае на бесконечном времени всё спокойно.
Ты имеешь ввиду сумма всех состояний каждой молекулы на очень большом отрезке времени? Так и Землю можно посчитать однородной, лол.
>если не ошибаюсь, то оно даже ДВС обходит
Таки обходит, ДВС как минимум от нескольких лошадок, все туpбины до 5 квт - мусоp.
Земля неизолированная. А так, во многих случаях энтропия растет при запутывании друг с другом частиц. Но конечно само понятие энтропии переосмысливается. Там больше про "потерю" информации. Сейчас эта область развивается, а я не физик, меня тошнит от дираковских обозначений, и всё только на собачьем.
Это мы уже к демону Лапласа переходим. В замкнутой(и равновесной) системе поидее энтропия должна быть постоянной. Честно говоря, КМ, с ее квантовым эфиром, неопределенностями в измерениях, запутанностями и прочими вещами, больше напоминает математическую магию: за основу взяты вычисления, а что там на самом деле происходит хуй его знает, это открыто сами ученые говорят. Стоит очередному уравнению не сойтись с реальностью, и начинается.
>Внутренню энергию ты тоже пальцем не ощутишь.
как раз пальцем ее и ощущают - называется тепло.
>Энтропиеметра не существует.
потому что энтропия не абсолютная величина как, скажем, температура или давление, это скорее описание эволюции системы. Она всегда относительна предыдущему состоянию конкретной системы.
>Внутренню энергию
Она же и потенциальной может быть. Если я положу тебе кусок тротила в руку а в другой пластелин. ты не сможешь с закрытыми глазами отличить.
В том-то и дело. Мы изобрели математику, и сами же ее не понимаем. И вместо того чтобы как-то оптимизировать ее для понимания нашим мозгом, мы еще больше уходим в дебри непонятно чего, иногда охуевая когда они не соответствуют реальности. А на что они собсно рассчитывали описывая процессы которые сами же и выдумали?
могут.
И то, и то - полная хуйня, количество улыбаний никак не влияет на старение.
Это называется проточный ТЯРД, больше инфы в гугле.
> Данное правило лично для меня не поддается объяснению, как и возможность существования демона Максвелла: если представить, скажем, море, и на нем большой(в сотни гектар) плот, состоящий из отдельных поплавков, каждый из которых относительно других связан механически так чтобы преобразовывать разность подъема в крутящий момент, затем все это передавая на некий единый вал, то такая система вроде как будет работать. Что мешает то же самое повторить на микроуровне - хз.
Коротко это не объяснить, но Фейнман норм расписал достаточно понятно, гугли по ключевой фразе "храповик с собачкой.
Ты об этом?: http://www.all-fizika.com/article/index.php?id_article=442
Получается проблема не в самом движении, а в "криворукости" демона, которого расшатывает это же движение?
Это доказательство не дает наглядного понимания т.к. мы привыкли видеть довольно точные макроскопические механизмы, и представить неработающий механизм из-за вероятных колебаний весьма сложно.
Dзять хоть вот эту часть:
>Предположим, что части нашего устройства идеально упруги. Когда собачка пройдет через конец зубца и сработает пружинка, собачка ударится о колесико и начнет подпрыгивать. Если в это время произойдет очередная флуктуация, вертушка может повернуться и в другую сторону, так как зубец может проскользнуть под собачкой, когда та приподнята!
Получается не каждое подпрыгивание собачки будет давать обратное движение. О резонансе здесь понятное дело речи идти не может, а значит статистически поворотов в одну сторону будет больше чем у другую.
>а значит статистически поворотов в одну сторону будет больше чем у другую.
Нет, ведь дальше всё посчитано:
>Шанс, что система накопит достаточно энергии ε, чтобы поднять собачку до края зубца, есть ехр(—ε/kТ). Но вероятность того, что собачка поднимется случайно, тоже есть ехр(—ε/kТ). Значит, сколько раз собачка случайно поднимется, давая храповику свободно повернуться назад, столько же раз окажется достаточно энергии, чтобы при прижатой собачке вертушка повернулась вперед. Выйдет равновесие, а не вращение.
> Но вероятность того, что собачка поднимется случайно, тоже есть ехр(—ε/kТ).
Но то что собачка поднимется не означает что это произойдет именно в момент обратного поворота колеса. А вот прямой оборот ее поднимет в любом случае: само колесо + случайные подъемы от тепла.
>А вот прямой оборот ее поднимет в любом случае
Вот это неверно, если в момент прямого поворота собачка будет падать на храповик, то потребуется больше энергии на то, чтобы её поднять (нужно будет погасить ещё её кинетическую энергию), чем если когда собачка просто лежит на храповике, то есть с она с той же вероятностью пресекает движение вперёд, с какой допускает движение назад.
>Следующими этапами будут: как сделать ее человеческого роста,
Тут ты был прав. Но этот процесс я примерно представляю. А вот тут:
>как сделать млекопитающим с молочными железами четвертого размера.
Уже был не прав.
Ок, то есть суть задачи сводится к более подвинутому механизму, исключающему данные факторы?
Такой механизм принципиально невозможен (хотя бы потому, что это вечный двигатель второго рода), доказательства упоминались в твоей же ссылке, гугл в помощь. По простому это можно объяснить так: что бы ты не построил, его будет шатать тепловым движением так, что он не сможет выполнять свою функцию.
Значит так я похоже забыл упомянуть что все мои примеры идеальны и абстрактны(хотя это и так должно было понятно), как тот же самый идеальный газ, который к реальности вообще не имеет никого отношения, в примере 1 да если он был бы в реальности были бы неоднородности концентрации(из броуновского движения), очень маленькая разница температур из-за того что частицы между собой соударяются и у некоторых кинетическая энергия больше у других меньше, очень маленькая разница давления - из-за того что вполне вероятно ( 0,000....001%) рождение маленькой области вакуума из-за того что частицы бесцельно двигаются туда-сюда. Плюс квантовая механика кидает сюда свои фокусы: квантовая неопределенность и туннелирование которые тоже рождают флуктуации неоднородностей. Но прикол то в том что я приводил пример в идеальных/абстрактных условиях где броуновского движения, квантовой неопределенности, квантового туннелирования и прочих флуктуаций неоднородностей нет. И это кстати означает, что максимум энтропии недостижим точно так же как и абсолютный нуль температуры. И кстати пока писал вспомнил, из-за чего тепловую смерть Выбеленной отправили на помойку, и это значит, что даже в моем идеальном, абстрактном примере всё равно будет то ли одна, то ли две разности потенциалов. Гравитация. Силы притяжения на границе системы и в её центре будут разными, и чем ближе к центру системы тем выше там давление от частиц со стороны границы системы. Из-за этого вроде даже в холодных космических туманностях рождаются горячие звезды.
>что информация таки материальна
Ну да - различная последовательность различных "квантов" информации (1 и 0, возбужденный и спокойный нейрон, и т.д.), который человек может интерпретировать как угодно. А квант "информации" - это квант энергии между стояниями материальных источников информации. В мозгу это разница энергии между возбужденного и спокойного нейрона, в компьютере разница энергии между намагниченном доменом железа и размагниченным доменом железа, в каком нибудь квантовом компьютере разница между возбужденным и основным стоянием атома. В примере Штырлец проходя мимо условного окна увидел три утюга, значит облавы не будет, если бы было два - значит готовится облава, "квант" информации - есть утюг, нету утюга - квант энергии - потраченая энергия на достать утюг и поставить его на подоконник или 0(не трогать утюг вообще). В азбуке Морзе фонариком - "квант" информации - есть световой сигнал, нету светового сигнала (пробел), время светового сигнала 1 секунда(точка) и время светового сигнала 2 секунды(тире) - квант энергии - энергия на создание светового сигнала и время его использования - 0, x и 2x кДж соответственно.
Только что проснувшийся >>26150-кун
Значит так я похоже забыл упомянуть что все мои примеры идеальны и абстрактны(хотя это и так должно было понятно), как тот же самый идеальный газ, который к реальности вообще не имеет никого отношения, в примере 1 да если он был бы в реальности были бы неоднородности концентрации(из броуновского движения), очень маленькая разница температур из-за того что частицы между собой соударяются и у некоторых кинетическая энергия больше у других меньше, очень маленькая разница давления - из-за того что вполне вероятно ( 0,000....001%) рождение маленькой области вакуума из-за того что частицы бесцельно двигаются туда-сюда. Плюс квантовая механика кидает сюда свои фокусы: квантовая неопределенность и туннелирование которые тоже рождают флуктуации неоднородностей. Но прикол то в том что я приводил пример в идеальных/абстрактных условиях где броуновского движения, квантовой неопределенности, квантового туннелирования и прочих флуктуаций неоднородностей нет. И это кстати означает, что максимум энтропии недостижим точно так же как и абсолютный нуль температуры. И кстати пока писал вспомнил, из-за чего тепловую смерть Выбеленной отправили на помойку, и это значит, что даже в моем идеальном, абстрактном примере всё равно будет то ли одна, то ли две разности потенциалов. Гравитация. Силы притяжения на границе системы и в её центре будут разными, и чем ближе к центру системы тем выше там давление от частиц со стороны границы системы. Из-за этого вроде даже в холодных космических туманностях рождаются горячие звезды.
>что информация таки материальна
Ну да - различная последовательность различных "квантов" информации (1 и 0, возбужденный и спокойный нейрон, и т.д.), который человек может интерпретировать как угодно. А квант "информации" - это квант энергии между стояниями материальных источников информации. В мозгу это разница энергии между возбужденного и спокойного нейрона, в компьютере разница энергии между намагниченном доменом железа и размагниченным доменом железа, в каком нибудь квантовом компьютере разница между возбужденным и основным стоянием атома. В примере Штырлец проходя мимо условного окна увидел три утюга, значит облавы не будет, если бы было два - значит готовится облава, "квант" информации - есть утюг, нету утюга - квант энергии - потраченая энергия на достать утюг и поставить его на подоконник или 0(не трогать утюг вообще). В азбуке Морзе фонариком - "квант" информации - есть световой сигнал, нету светового сигнала (пробел), время светового сигнала 1 секунда(точка) и время светового сигнала 2 секунды(тире) - квант энергии - энергия на создание светового сигнала и время его использования - 0, x и 2x кДж соответственно.
Только что проснувшийся >>26150-кун
Все зависит от pазмеpа.
Да и не у каждой туpбины кпд будет больше, у той же газовой туpбины он меньше.
Все из-за огpомной скоpости, скоpость с котоpой газ истекает из сопла - огpомная, значит колесо должно вpащаться с огpомной скоpостью, либо быть не менее 3-х метpов в диаметpе.
У туpбин Тесла, кстати, такая же пpоблема, лопатка туpбины должна двигаться с половинной скоpостью паpа.
Но я тоже тут чую подвох, ибо pаз все pадужно, то давно могли бы делать pеактивы для моделек на тесле дешево, но ничего нет пока.
>то давно могли бы делать pеактивы для моделек на тесле дешево, но ничего нет пока
А компpессоpы у него хуево pаботают.
https://www.youtube.com/watch?v=xdZVx07U5SU
Лол. Баянище. Два года назад в интернетах писали про этих собирающихся китайцев. Так до сих пор и не собрались. А секрет прост - китайцы не собирались. Это утка.
Да ладно, хуй с ними с китайцами, вроде наса всякие опыты проводила и говорит что вроде как работает. Мне больше интересны примеры, которые я привёл.
Почему не имеет-то? Имеет. Свет летит от корабля унося с собой импульс, поэтому для его сохранения корабль будет двигаться в другую сторону.
Так ведь вроде бы вся суть ведра в том, что ему не нужно рабочее тело, не? Запасись энергией и лети куда хочешь.
Тащемта pаботают хуево, потому что обоpотов мало. Но в любом случае хуже, чем pадиальный компpессоp
На тыльную область коры.
Cвет это тоже рабочее тело.
Со стороны химии не скажу, но вот со стороны физики всё зависит от того, какой процесс рассматривается. Например при поляризации кристаллических диэлектриков - кристалл рассматривается как одна большая молекула. Но при, например, рассмотрении распространения волн в среде - принимать всё вещество за одну молекулу уже нельзя.
Откуда ты знаешь? Может они уже проверили, наткнулись на подтверждение и решили по-тихому развивать технологию, дабы обогнать технически остальных.
> Почему негры черные
Из-за большого количества меланина в коже, который поглощает ультрафиолет и этим самым защищает более глубокие слои кожи
Лол, один поехавший из РФ уже собрал денег и оплатил подобный эксперимент с ведром в космосе. Никакой тяги нет.
Инерциоид это не ведро.
Опыт проводился 1 раз на земле внутри изолированной от внешних воздействий стальной вакуумной трубы. Почему-то не измеряли возникающее магнитное поле и, соответственно забили хуй на магнитное взаимодействие между трубой и помещённым внутрь него ведром.
С таким же успехом можно засунуть вместо ведра обыкновенный электромагнит. Хотя нет, не с таким же. От электромагнита тяга была бы в разы больше.
Но вот какая незадача - космос - это тебе не труба.
Ты не видишь. Ты опознаёшь элементы.
А ты от куда знаешь?
Потому что слово "негр" происходит от португальского "негро", что в переводе на русский язык означает "чорный".
На самом деле негры не чорные.
>Им не должно быть жарче?
На самом деле нет. Закон Кирхгофа (это физика) - чем чернее тело, тем быстрее оно выпускает тепло из себя.
Чиво блядь?
> Установил один из основных законов теплового излучения, согласно которому отношение испускательной способности тела к поглощательной не зависит от природы излучающего тела
>>26422
Потому что в их коже специальные черные хуевинки, которые поглощают вредное излучение, не позволяя ему проникать в глубь тканей. А черное внезапно лучше поглощает излучения.
Ты нихуя не видишь. Твой мозг сугубо абстрактно интерпретирует ничтожную часть взаимодействующих с твоими органами процессов.
Нужно мнение анона, не понаслышке знакомого с железнодорожной инфраструктурой.
Сейчас железная дорога применяется для поездок между городами повсеместно. В то же время, более современным её аналогом и потенциальным убийцей станет монорельс, обладающий нехилым рядом преимуществ.
Суть вопроса заключается в следующим:
А может ли монорельс заменить железную дорогу в масштабах страны? Имеются ли у него какие-то ограничения в дальних путешествиях, которые не позволят применять его таким образом?
>>26451
>Чиво блядь?
Чтобы разобраться в таком вопорсе, нужны уточнения:
1. Что за страна? Каковы её размеры? Каковы расстояния между крупными городами?
2. Какой протяжённости должна быть дорога, чтобы можно было однозначно утверждать: да, такая дорога в масштабах страны ?
3. Какой средний пассажиропоток на единицу времени между крупными городами?
1. Предположим, Северная Америка. 24 миллиона километров. Среднее расстояние будет где-то 1000 км между городами. Где-то меньше, где-то возможно больше.
2. Такой, чтобы охватывала все крупные города внутри страны, позволяя добраться до них с помощью монорельса.
3. Вот этого не знаю. От балды, наверное несколько тысяч человек в день.
я спрашиваю о внутренних причинах. одни понимают, а другие нет. какова разница в мозгах/нервных системах?
Ну потому что это московский монорельс. Как будто тебе неизвестно, что у нас всё через жопу. Можешь посмотреть на Китай и Японию. Там нормальные монорельсы.
>крупные города
Крупный город - это сколько (в цифрах) ?
Если говорить об обычном, близком к среднестатистическому быдле, то у таких склонность к математике определяется социальной мотивацией.
А если рассматривать особо выдающихся личностей, осиливающих что-то более сложное, чем линейные уравнения, то тут надо иметь в виду, что среди мальчиков гораздо больше умственных отклонений как в положительную, так и в отрицательную сторону, по сравнению с девочками.
Иными словами: среди мальчиков чаще встречаются как дибилы, так и гении. С дибилами всё понятно - они прсто дибилы и не могут ни во что.
А гении на то и гении, что могут во что угодно, хоть в математику, хоть в литературу. Внутренние механизмы гениальности до сих пор не раскрыты. Единственное, что известно -- у умных родителей часще рождаются гении, у дибилов гении не рождаются никогда.
Ну и в далее у таких вот гениев склонность к математике задается так же, как и у обычного быдла - социальной мотивацией.
Ок. Значит надо соединить миллионные города Северной Америки. Такие города расположены не слишком на севере и не слишком на юге. Климатические условия входят в тот же диапазон, в котором существуют ныне действующие короткие монорельсовые дороги.
>>26460
Стоимостью пренебречь.
Ответ: технических ограничений нет.
В больших масштабах она не нужна.
Но вот, если pазмеp до киловатта, то кпд очень неплохой, у паpового двигателя кпд в любом случае маленький.
Всегда было интеpесно, что будет, если в ней сделать не одно сопло, а 16, 32? Недавно видел такую туpбину на 4 сопла, там за 1 секунду 20к обоpотов, неужто кpутящий момент pастет?
https://www.youtube.com/watch?v=5hldFLU8Aig
Нахуй ты мне какого-то мошенника пpинес?
Или на все с названием "Тесла" у тебя pефлекс как на фpичество?
Благодарю, анон.
Пытались. Получился пикрелейтед.
Не взлетело, т.к. без понятия времени нельзя сформулировать причинно-следственные связи и вместо упорядоченных законов природы получается хаос не связанных между собой явлений.
Потому что говно не содержит полезной ценности для организма.
Вообще надо понимать, что понятие "вонь" относительно. Для тебя это может быть вонь, а для собачки - приятный аромат.
Даже если бы времени не было, эту величину надо было бы ввести.
Попробуй без времени определить, столкнутся или пролетыт мимо друг друга объекты на пикрелейтед. Вангую, что у тебя не получится.
Хм, а что если электрическое поле - это на самом деле и есть вектора силы, которые действуют на сам заряд, но так как этот заряд находится в центре (является их источником), то силовые вектора, исходящие в одном направлении, уравновешены векторами в другом?
Что за график?
>>26511
>Т.е. такой эффект, при котором молекула отдала бы больше энергии, чем имеет. Считается, что это не возможно.
А нарушение закона сохранения энергии, тогда ясно. (Хотя при химических реакциях реагенты вначале имеют около комнатную температуру а продукты в конце температуру выше комнатной)
>при химических реакциях реагенты вначале имеют около комнатную температуру а продукты в конце температуру выше комнатной
Угу, и при этом исходные молекулы перестают существовать.
Тут вот что происходит. Предположим космонавту придали импульс, которого хватило бы на десять световых скоростей. Собственно, до звезды, расстояние до которой 30 световых лет, он бы долетел за три года. Ну, для себя. Вот только сторонний наблюдатель решил бы, что он двигается со скоростью близкой к световой (снизу), и поэтому сторонний наблюдатель решил бы, что космонавт летел 30 лет, а не 3. Поэтому космонавт (который тоже бы не подумал что двигается со сверхсветовой скоростью - для него бы все расстояния стали бы в десять раз меньше) смотался туда-сюда за 6 лет, а наблюдатель ждал бы его 60 лет. Такие дела.
>это магия?
Не, магия - это то,что не поддаётся математическому расчёту.
В случае релятивистского замедления времени мы имеем дело с вполне просчитываемым эффектом. Поэтому это не магия, а закон природы.
Релятивистское замедление это которое для ИСО, по-моему, а как объясняется, почему ускорение замедляет процессы?
>почему ускорение замедляет процессы?
Единственный человек, который возможно знал ответ, умер. От него остались только математические формулы.
А дальше учоные решили не разбираться и придумали отмазку "антропный принцип".
https://ru.wikipedia.org/wiki/Антропный_принцип
>>26527
Не-не-не! Там лютый матан. Ебись сам с матаном.
Я пришёл сюда отвечать на тупые вопросы, а не интегралы вычислять.
А что блядь, если лифт наэлектризовать и он будет с ускорением двигаться к противоположно заряженному телу, то может тогда и электромагнитного поля никакого нет, и на самом деле какие-то искривления пространства возникают вокруг зарядов?
В общем меня эта тема нереально бесит.
За Вами выехали
Сделать лучше должны преподаватели и научно-популярные каналы, который все время пересказывают байку про лифт.
У световой частицы нет массы покоя. Поэтому просто так свет не притягивается. Однако гравитации сопутствует искажение пространства. Свет, не зависимо от того, частица ли он или волна, в пространстве ведёт себя как движущийся объект. Т.к. пространство искривляется, то и траектория света в этом постранстве тоже искривляется.
>придали импульс, которого хватило бы на десять световых скоростей
с этого момента подробней. Возможно ли движение со скоростью, превышающей скорость света?
Сейчас упарываю Фейнмана, но ощущение, что его мало.
> А теперь в своем тексте поменяй местами стороннего наблюдателя и космонавта.
Нельзя менять, ускорение - величина абсолютная, космонавт со своим кораблём ускоряется, наблюдатель со своей планетой - нет.
Электромагнитное поле квантуется, гравитация пока нет. Электромагнитное поле не вызывает прецессию из-за искажения пространства, гравитационное - вызывает, что можно наблюдать у Меркурия.
Смотри, тут весь прикол в релятивистских изменениях времени и расстояний. Движение со скоростью выше c невозможно, но когда тело движется со скорость, близкой к c, то в его системе отсчета все расстояния сокращаются. То есть для космонавта звезда будет приближаться со скоростью меньшей скорости света, но само расстояние до неё будет гораздо меньше.
Итог: космонавты вполне могут прилететь "для себя" очень быстро, но дляоставшихся дома землян это займёт тыкву хучу времени.
Неверно, свет имеет энергию, которая как известно эквивалентна массе и поэтому гравитирует, если бы было как ты говоришь, то гравитирующее тело не притягивалось бы к пролетающему мимо него свету и это нарушало бы закон сохранения импульса.
Проиграл с тыквы времени.
Лол, ему вообще похуй как ты его разместишь относительно планеты, главное чтоб ротор и статор располагались правильно относительно друг друга.
>чиво
Просто я не вспомнил такого закона, да и вообще не слышал, что черное тело быстрее излучает тепло. И даже если это так, к работе меланина относится разве что косвенно.
Многие посты здесь пишутся невысыпающимися школьниками, либо в приступах шизофазии, сходу и не разберешься. И пруфни, что черное тело быстрее "выпускает тепло", пожалуйста.
Спасибо.
>И пруфни, что черное тело быстрее "выпускает тепло", пожалуйста.
Мимо другой анон, но пруфну:
>Отношение излучательной способности любого тела к его поглощательной способности одинаково для всех тел при данной температуре для данной частоты и не зависит от их формы и химической природы.
То есть, если тело всё поглощает (то есть оно чёрное), то оно и излучает не хуже, соответственно тела, которые хуёво поглощают (то есть они вместо этого отражают, белые/зеркальные), и излучают хуёво, то есть хуже, чем чёрные тела. В любом случае это следует из второго начала термодинамики, если бы вышесказанное было неверно, то можно было бы передавать тепло от более холодного тела к более горячему.
Но тот анон был неправ в отношении негров, потому что поглощают они в видимом (а также немного в ультрафиолетовом спектре), а излучают в инфракрасном, то есть в инфракрасном спектре они вполне могут быть и не чёрными (точно не знаю) и излучать гораздо меньше, впрочем они в любом случае излучают меньше, потому энергия инфракрасного излучения намного ниже. Да ещё и солнце намного горячее негров, поэтому они не могут излучать столько же, сколько поглощают, так что неграм действительно жарче от солнца, чем белым людям.
>неграм действительно жарче от солнца, чем белым людям
Ну-ну.
Только на практике оказывается, что негры лучше переносят солнцепёк, чем белые.
Не работает твоя теория, однако.
Я просто оставлю это здесь http://teslatech.com.ua/biblos/disks_tech/eksperem_issledovanie_diskovoy_mikroturbini.pdf
Кстати, у них получился кпд ~50% http://www.business-vector.info/прорывной-проект-энгельсских-иннова/
Переносят лучше они по другим причинам.
Как можно представить элементарные частицы? Вот допустим есть антенна и переменный ток, как определить кол-во колебаний или еще чего чтобы выделить их этого всего 1 фотон?
Или вот еще пример: берем веревочку, привязанную к чему-то, дергаем, по ней побежала волна. Тут поидее где-то должен быть фонон, как его выделить?
>Как можно представить элементарные частицы?
как угодно, увидеть их все равно нельзя.
>чтобы выделить их этого всего 1 фотон?
если взять оптическое излучение, то можно сделать систему фильтров, через которые будет пролетать только один фотон. Также как есть и счетчики фотонов.
Двач, а как посчитать количество обрабатываемых поверхностей? С меня тонны нефти
> увидеть их все равно нельзя.
но волну можно увидеть или нарисовать. Осталось только выделить из нее 1 фотон.
Минутка суровой реальности: а внезапно нет никаких эл. магнитных волн, полей, да и фотонов тоже. Это математические и физические модели.
>но волну можно увидеть
Какую? На воде или в веревке, да, можно, пока свет на них падает и отражается. А электромагнитное взаимодействие между объектами, в виде некой сущности слоника, вылетающей из одной точки и прилетающей в другую, увидеть нельзя, вообще. Один атом энергию потерял, другой получил, для стороннего наблюдателя в пустом пространстве между ними не происходило ровным счетом ничего, если не становиться участником этой системы.
Есть такие двигатели: PMDC (двиг. пост. тока), BLDC (бесщёточный двиг. пост. тока), PMAC (синхронный бесщёточный двиг. перемен. тока). Я правильно понимаю, что последние два типа двигателей очень похожи, а механические характеристики у всех трёх типов двигателей одинаковы? (если нет, то скиньте, я так и не нашёл в инете, интересует тупо зависимость крутящего момента от оборотов, пиковый и постоянный моменты)
>На воде или в веревке, да, можно, пока свет на них падает и отражается
>точно так же антеннами и датчиками можно определить напряженности полей в различных местах, получив примерную картину распространения э/м волны. Можно определить, скажем, длину волны, нарисовать ее макет. А вот размеры фотона, соотношение его с волной(сколько периодов содержит, например) - не понятны. И ладно там еще поле условно простирается до конца Вселенной(в бесконечность, да), но фотон это ж вроде как частица, а частицы механически имеют ограниченный размер, массу(в данном случае релятивистскую) и все такое.
То есть не существует никаких доказательств того, что человеческое сознание оказывает на реальность какое-то влияние, которого не могло бы существовать без него. Я все правильно понял?
yep
Это просто эффект, который интуитивно кажется таким, но на самом деле всё раскладывается довольно просто.
Судя по ответу выше, сама квантовая физика существует только в человеческом сознании, и то в виде формул. Что там в реали хуй знает, физикам главное абстрагироваться чтоб уравнения сошлись.
Спасибо, анон.
Вообще хотелось верить в то, что будучи человеком, это делает меня каким-то особым во вселенной. Тонкая настройка вселенной, квантовые теории сознания фон Неймана, вот это все. Но с другой стороны, хочется принимать реальность такой, какой она есть.
Если в человечестве нет ничего особенного, то может быть однажды мы откинем в прошлое наши полные страхов и боли тела и заменим человечество созданными нами ИИ, которые будут изучать реальность и принимать ее лучше нас.
Увидели молекулу как шарик - заебись. Я вот выше спрашивал как выглядел бы фотон, пошли варианты что его вообще нет.
>Как может блять разрушиться что-то, скованное льдом?
Во первых ледник оказывает давление, во вторых - на сколько я понимаю, они обладают подвижностью. Таким образом, то, что оказывается зажатым в льдах, в течении тысяч лет плавно перемалывается до состояния пыли (но это не точно).
Не совсем понимаю, ведь это происходит не так, что с севера приезжает километровая глыба льда и перемалывает под собой всё, а постепенно территория покрывается слоем льда. Как у нас зимой постепенно всё больше снега наваливает. Или что-то другое?
Ну я лично себе это так представляю - вот есть город из говна и глины, наступает оледенение региона. Все покрывается снегом и постепенно наращивается слоем льда. Этот слой льда становится все тяжелее и ломает под собой все, что не в состояние выдержать его вес. А потом эти ледники еще и начинают двигаться (for whatever reason) и превращаются в настоящую молотилку, перемалывая все, что находится под их слоем.
Вот что-то на тему движения ледников:
https://www.youtube.com/watch?v=RnlPrdMoQ1Y
>квантовая физика существует только в человеческом сознании
разумеется
>и то в виде формул
безусловно, иначе не получается
> Что там в реали хуй знает
и даже он не знает
>чтоб уравнения сошлись.
естественно, тогда все это будет работать
На ютюбе недавно вышел цикл лекций Чирцова по физике, можешь навернуть, если не гуманитарий.
Ну во первых, городов тогда не было.
Во вторых, культура не существует без носителей, т.е. без человеков. Есть человеки - есть культура. Нет человеков - нет культуры. Таким образом разрушение культуры возможно только одним способом - смертью человеков.
Во вторых, оледенение происходило постепенно, не за 1 сезон, а в течение десятилетий или веков. За это время в европе человеки успевали съебаться на юга и унести свою культуру с собой.
Вот там на югах и происходили разрушения культур. Культуры разрушались войнами. И тут как повезёт. В одних случаях понаехавшие с севера проигрывали войну и тогда разрушалась культура северян. В других случаях понаехавшие побеждали и тогда сохраняли свою культуру. Но чаще всего происходило смешивание культур понаехавших и местных и образовывались новые культуры. Т.е культура не разрушалась, а перерождалась в другую культуру.
Лол. Сначала надо понять прикол Гамильтоновой классической механики. И только потом курить кванты.
если бы ты посмотрел названия лекций, то обратил внимание, что квантмех там в конце. И вообще все крайне упрощенно, школьных знаний для входа достаточно.
Думаю что металлический, т.к. любой металл это по сути твой куб свободных электронов, скрепленный ядрами атомов.
Лол. Это можно посчитать кстати. Только сейчас об этом подумал. Причем наверное хватит листка бумаги.
Если их сложить много в одном месте, то да.
>куб со сторонами, скажем, 5 см
Зависит от того, какое расстояние считать минимальным. В теории должна получиться 5см чёрная дыра.
Но ты ввёл приблизительное число электронов
>миллиарды и миллиарды
Это порядка 1010. Если такое число электронов сжать вместе до минимальных расстояний между электронами, то размер объекта будет нанометровый. Т.е. 5см не получится.
Твоя задача построена на взаимоисключающих параграфах, поэтому бессмысленна даже в теории и не имеет ответа
Зависит от количества таблеток и их размера. Ты не указал ни количество, ни размер. Задача не имеет решения.
Это я и хотел узнать. Есть ли какая-то связь? Или может быть? Или точно нет?
Эйнштейн сравнивал гравитацию с падающим лифтом - мол, мы все скатываемся в центр гравитации. Если мысленно провести такой же эксперимент с магнитным или электрическим полем, при притяжении к магниту в металлическом лифте металлические предметы так же будут ощущать невесомость. Значит ли это что электромагнитное поле так же искривляет пространство?
Значит. Правда, это искривление уже не пространства-времени, а пространства главного U(1)-расслоения пространства-времени. То есть электромагнитное поле (тензор Фарадея) - это аналог тензора кривизны, но не для связности Леви-Чивиты, а для связности главного расслоения.
Была одна теория, в которой ЭМ поле "искривляет" именно "реальное" пространство (теория Калуцы-Клейна), но там вводится дополнительное измерение. Современный взгляд такой, который я изложил выше. Остальные взаимодействия тоже описываются подобным образом (только группа U(1) меняется)
Организация тепловой энергии в негэнтропийное упорядоченное движение в процессе самоорганизации в наноструктурах.
Что-то типа вечного двигателя второго рода, но во второй его формулировке:
Вечный двигатель второго рода — машина, которая, будучи пущена в ход, превращала бы в работу всё тепло, извлекаемое из окружающих тел.
Если так, то второй закон термодинамики здесь неприменим, так как у вечного двигателя второго рода есть ещё одна формулировка:
Вечный двигатель первого рода — устройство, способное бесконечно совершать работу без затрат топлива или других энергетических ресурсов.
Здесь же, тепло извлекается из окружающей среды, и если это реально работает, то можно назвать это холодильнико-генератором.
Алсо, тупых вопросов обо всем тред.
Тоже об этом думал. Было бы охуенно, если б полдня работала одна половина мозга, полдня - другая.
>Но ты ввёл приблизительное число электронов
Ничего я не вводил. Допустим, мы смогли собрать много электронов и сжать их, но чтобы расстояние между ними было больше радиуса Шварцшильда. Собрали столько, что получился тот же куб. Вопрос в силе.
Тебе надо было родиться дельфином:
Дело в том, что все высшие животные (птицы и млекопитающие, включая человека) не только обязательно спят какую-то часть суток, но и в наиболее глубоких стадиях сна полностью расслабляются, теряют подвижность.
Для дельфина это невозможно — обитатель моря, он дышит воздухом, и потеря подвижности грозит ему тем, что он не поднимется к поверхности воды для очередного вдоха, захлебнется. Давно замечено, что дельфины никогда не замирают в полной неподвижности, они всегда хотя бы слегка двигаются и поднимаются на поверхность для дыхания. Так спят ли они вообще и если да, то как и когда?
Существовало много предположений по этому вопросу. Но единственным надежным способом решить вопрос было изучение биоэлектрической активности их мозга, по которой можно уверенно сказать, когда животное бодрствует, а когда спит. Такие исследования были выполнены. Результаты оказались неожиданными.
До сих пор считалось само собой разумеющимся, что когда сон сменяется бодрствованием или бодрствование сном, то эти изменения происходят во всем мозге — ив правом и в левом его полушариях: ведь два полушария мозга отвечают за работу двух половин нашего тела. Именно так обстоит дело у человека и у всех животных, у которых до сих пор исследовался сон. Но у дельфина все происходит иначе. Два полушария мозга дельфина спят но одновременно, а поочередно: когда одно спит, другое активно.
Потом они меняются ролями, и то полушарие, которое было активно, засылает, а «выспавшееся» бодрствует. Оно и обеспечивает управление телом дельфина, необходимое, чтобы он нормально дышал и не захлебнулся. Естественно, есть время, когда бодрствуют оба полушария
Не совсем. То есть если их два, то да. Одно "творческое", другое "рациональное". Но если вырезать бОльшую часть одного полушария (кроме гипофиза и еще чего-то), то второе берет его обязанности на себя.
Это надо считать. При таких значениях Шредингером уже не воспользуешься, а надо через уравнение Паули. Лениво и я ничего им не считал и не умею.
Да это заголовок говно. Этот графен просто тепло в электричество конвертирует. В этом нет ничего экстраординарного, но это может быть очень полезным.
Подзарядка кардиостимуляторов, диализных минимашин или бионических протезов от тепла человеческого тела.
Новые радиаторы для микросхем, позволяющие часть выделяемого тепла вернуть в виде электричества.
Приложений реально дохера. Может это даже заменит ставшее нам привычным колесо и воду в большинстве типов электростанций.
Это называется Демон Максвелла - некая сущность, способная без затрат энергии(!) переносить тепло их холодного тела в более горячее.
Если с вечным двигателем вроде разобрались - энергия материальна, не появляется и не исчезает никуда, то второй просто так не понятен, как для понимания так и для различных теорий. Впрочем, в квантмехе и первый-то не особо соблюдается.
Второй закон не просто описывает работу и тепловую энергию, а чуть ли не делает материальной информацию.
>Подзарядка кардиостимуляторов
Внутри организма так-то перепадов температуры нет, только нарушать второй закон термодинамики.
Хорошо, давай проще.
Не всегда всё действо физики описывается в собственно реальном пространстве. Фактически, оно наделено всего несколькими симметриями (относительно сдвигов в пространстве, сдвигов во времени, поворотов в пространстве, и геперболических поворотов в пространстве-времени, гугли группу Ллренца или группу Пуанкаре для афинного пространства).
С другой стороны, уже в классической механике вводится фазовое пространство, ну а в физике частиц есть всякие спциальные заряды и изоспины, которые выражают "внутренние" симметрии каких-то специальных пространств, связанных с частицами.
Возвращаясь к электромагнетизму, ещё до "современной" фмзики (относительность/квантовая) обгаружили симметрию кравнений Максвелла (гугли калибровку электромагнитных потенциалов, вкраце - к э/м потенциалам можно прибавить определенные слагаемые и уравнение Максвелла всё равно будет выполняться, т. е. потенциалы определены неоднозначно).
Много позже узнали, что уравнение Шрёдингера из квантовой механики тоже обладает симметрией (локальной калибровочной инвариантности) из-за того, что, на пальцах, в конце мы возводим волновую функцию в квадрат и комплексная фаза "пропадает".
Так вот оказывается, что для соблюдения этой симметрии нам нужно ввести специальное внешнее поле, которое по чистой (не совсем) случайности можно отождествить с реальным электромагнитным полем! Это на самом деле охуенно интересная вещь. Симметрия называется U(1) т. к. она совпадает с симметрией окружности, которая описывается унитарными матрицами размерности 1х1.
Блядь, как мне ещё объяснить калибровочные теории Янга-Миллза на пальцах?
iq ничего не определяет, я в хорошем настроении получу один результат, а в плохом - другой. Получается iq определяет настроение лол? Это как если бы твой рост менялся по неведомым причинам. Не говоря уже о том, что его прохождение можно натренировать или угадать случайно.
1. В хорошем собранном настроении, без стимуляторов.
2. На детекторе лжи в анусе спрашиваем, пытался ли пройти ли тест за прошедшие три года и задрачивал ли вообще когда-либо активно. Экстерминируем непрошедших. Думаю, психолухи всё-таки выработали более-менее достоверную методику
Короче есть пятое измерение пространства - электромагнитное и вот оно то и искажается от электромагнитных воздействий точно также, как обычные четыре искажаются от гравитации.
Вот смотрите: в идеальном космосе, где ничего больше нет, помещает 3 одинаковых тела, на вершины равностороннего треугольника. Располагаем над этой системой наблюдателя, он видит, как тела падают в центр этого треугольника, проходит Х времени.
Теперь предаем импульс телам, как на пике, только не борщим, что бы наши тела не разлетелись нахуй, а наблюдателя раскручиваем вокруг своей оси. Ну что бы вся система вращалась синхронно, вокруг оси (центр треугольника - наблюдатель).
И что же изменилось для нашего наблюдателя? Да нихуя, кроме того, что наши тела не падают в центр, точнее делают это медленно.
Как объяснит наблюдатель, причину такого замедления, если он сам не знает, что вращается, ориентиров нет, скорости он не чувствует.
Шах и мат, Эпштерн! Когда я введу капчу и нажму ентр, вселенная схлопнется!
Не схлопнулась, потому что пикче забыл, вот щас точно схлопнется.
Собака зарыта именно здесь:
>если он сам не знает, что вращается
Идеальный наблюдатель всегда "знает" - если он вращается/движется с ускорением/испытывает действие эквивалентной силы притяжения. (Вращение - частный случай движения с УСКОРЕНИЕМ.)
Например, на планете у которой на "небесной сфере" - нет никаких объектов для привязки - вращение самой планеты, можно заметить, все равно.
Действительно, обосрамс.
Обычно подразумевают софт, но есть и аппаратные (https://ru.wikipedia.org/wiki/TrueNorth)
> Если железо, то чем отличаются от обычных суперкомпуктеров
Архитектурой
> если совтвэйр, то чем отличаются от обычных программ
Ничем. Разве что, для их обучения обычно используют видеокарту
Ну 5-ое, 6-ое и 7-ое, окей.
Спасибо
Тренировка сети на тестовых данных. Например, нейросетке скармливают тысячи фотографий лиц, и она постепенно учится выделять признаки человеческого лица
https://ru.wikipedia.org/wiki/Искусственная_нейронная_сеть#Обучение_сети
тут нихуя не поймет это, ну ты лол такое постить
Ну даже если заставить пациента проходить эти тестики чтобы он набил статистику честную, всё равно результат будет говорить только о том насколько он хорошо эти тестики проходит. Они же все построены на нахождении закономерности и вычислении лишнего. И даже этот навык такими тестами ты не проверишь. Может ты на картинках легко это делаешь, а в реальности не можешь найти закономерность в событиях.
Совсем уж даунов которые пускают слюну на этот тест конечно можно определить, но это и так видно.
тогда всем пизда
И всё-таки они считаются лучшим из того, что есть не?, да и разные они бывают; далеко не во всех заданиях оптимально действовать логическим брутфорсом. Не всякий набравший 130 умён, но почти любой, кого считают умным не из жополижеских мотивов, эти 130 легко наберёт. А 150+ действительно мало что об уме говорит, больше процент снобствующих пидрил определяет.
Обычная беседа специалиста с пациентом намного точнее тестика. Это как и психологические тесты - созданы для обработки большого количества людей. Точностью там и не пахнет.
Ребзя такой вопрос.
Если на велик поставить ветряк (лопасти) и от него механически, через понижающий редуктор, подать вращение на колесо. Поедет ли велик при встречном ветре?
2. Какие области наиболее мозга развиты у художников?
Маленькие капельки воды. Очень сильно рассеивают свет.
На парах сделал посев в эндо-агар, но ничего не выросло, у других все норм было. Делал 2 раза, и безуспешно, чашка со средой оказывалась идеально чистой. Возможно ли это из-за того, что я болею (на момент тех пар был в инкубационном периоде, получается) и надышал случайно хуйней из носа, которая повыкашивала палочки?
Еще помимо среды эндо была другая, которая по идеи должна была выявить чувствительность к антибиотикам. Так вот тоже ничего не возросло
ответ на первый вопрос - надо меньше сидеть в /b/ и /po/
https://pp.userapi.com/c840622/v840622346/308a3/PZVJZgq1Z-w.jpg
Аноним, почему эта фича не должна работать?
Первое, что приходит в глову, так это то, что вода под своим весом - просто выльется снизу - из-за разности давлений...
но если конструкция вращается, то она не должна интенсивно выливаться, так как проталкивается....
Если вода постепенно будет подаваться сверху, то система должна работать.
Понятное дело, что при малой утечке воды внизу, эта система должна быть намного эффективнее,
чем использование лопастей в случае подачи воды, если крутить ею эти колёса в обратную сторону.
Так вот, почему бы не изогнуть трубку буквой J и не опустить её в водоём,
чтобы вверх заходили какие-нибудь легкие пенопастовые шарики, и выталкивались водой.
Сперва думается, что из-за сообщающихся сосудов трубка, изогнутая буквой J будет сразу наполнена водой
из-за разности давлений снизу и сверху.
Но если шарики будут выходить из неё, они должны будут выталкивать эту воду,
и от интенсивности выхода этих лёгких предметов снизу трубки
может зависеть количество воды, выталкиваемой из днища трубки,
а значит и глубина погружения и размер конструкции.
Я думаю эта система будет работать, потому что это не вечный двигатель, и по-сути в такой системе работу выполняет планета,
притягивая воду и вдавливая эту воду вниз - а сила Архимеда просто следствие из этого.
Но если разность давлений сверху трубки и снизу - стремится вдавить воду в днище трубки,
то эта разность давлений может вдавить и сами эти лёгкие поплавки - тогда всё встанет... Ок.
А что если внутри поплавков сделать маленькие дырочки с клапанами,
через которые вода просачивалась бы чуток внутрь трубки,
и которые пропускали бы воду чуть-чуть в днище трубки,
выталкивая тем самым поплавок (он же легче),
но закрывать эти дырочки и клапана на других поплавках если вода слишком далеко идёт.
Короче сделать что-то типа пенопластового кирпича с дырочками, но дырочками не простыми, а с клапанами,
ну чтобы вода под давлением открывала эти клапана и просачивалась, но не во всю трубу, а до определённого уровня.
А дальше уже чтоб поплавок выталкивался и ускорялся.
Можно так вообще? Если нет, то почему?..
Аноним, почему эта фича не должна работать?
Первое, что приходит в глову, так это то, что вода под своим весом - просто выльется снизу - из-за разности давлений...
но если конструкция вращается, то она не должна интенсивно выливаться, так как проталкивается....
Если вода постепенно будет подаваться сверху, то система должна работать.
Понятное дело, что при малой утечке воды внизу, эта система должна быть намного эффективнее,
чем использование лопастей в случае подачи воды, если крутить ею эти колёса в обратную сторону.
Так вот, почему бы не изогнуть трубку буквой J и не опустить её в водоём,
чтобы вверх заходили какие-нибудь легкие пенопастовые шарики, и выталкивались водой.
Сперва думается, что из-за сообщающихся сосудов трубка, изогнутая буквой J будет сразу наполнена водой
из-за разности давлений снизу и сверху.
Но если шарики будут выходить из неё, они должны будут выталкивать эту воду,
и от интенсивности выхода этих лёгких предметов снизу трубки
может зависеть количество воды, выталкиваемой из днища трубки,
а значит и глубина погружения и размер конструкции.
Я думаю эта система будет работать, потому что это не вечный двигатель, и по-сути в такой системе работу выполняет планета,
притягивая воду и вдавливая эту воду вниз - а сила Архимеда просто следствие из этого.
Но если разность давлений сверху трубки и снизу - стремится вдавить воду в днище трубки,
то эта разность давлений может вдавить и сами эти лёгкие поплавки - тогда всё встанет... Ок.
А что если внутри поплавков сделать маленькие дырочки с клапанами,
через которые вода просачивалась бы чуток внутрь трубки,
и которые пропускали бы воду чуть-чуть в днище трубки,
выталкивая тем самым поплавок (он же легче),
но закрывать эти дырочки и клапана на других поплавках если вода слишком далеко идёт.
Короче сделать что-то типа пенопластового кирпича с дырочками, но дырочками не простыми, а с клапанами,
ну чтобы вода под давлением открывала эти клапана и просачивалась, но не во всю трубу, а до определённого уровня.
А дальше уже чтоб поплавок выталкивался и ускорялся.
Можно так вообще? Если нет, то почему?..
Смотрим на первую картинку, на самый нижний блок. На него с одной стороны действует атмосферное давление, а с другой- атмосферное плюс давление столба воды. Если вспомнить, что сила Архимеда - это по сути разность давлений между верхней и нижней гранью блока, то можно заметить, что сила, которая выталкивает нижний блок из воды, больше, чем сила Архимеда для всех остальных блоков. Это на пальцах, если посчитать аккуратно, то получишь, что на то, чтобы запихать блок в воду, необходимо потратить ровно столько же энергии, сколько этот блок даст, всплывая.
Потому что соударение абсолютно неупругое. При таком контакте часть энергии теряется на упрощенно деформацию тел (и уходит в тепло, соответственно).
Годно. Действительно на нижний блок действует максимальное давление.
Идея с дырочками и клапанами - накрылась медным тазом.
Потому что малое количество зашедшей воды не обеспечит то давление снизу,
необходимое для того, чтобы таки-вытолкнуть поплавок из трубы.
Для этого надо заполнить трубу хотя-бы до уровня сообщающихся сосудов,
а туда уже хрен воткнёшь другой поплавок. =)
А есть ли принципиальная возможность существования жизни, которая будет хранить "генетическую информацию" по другому принципу (не "кодирование с помощью цепочечной молекулы")?
В каком-то журнале была статья про взвесь заряженных пылинок в особых условиях, которые собирались в цепочки, и в окрестностях одной такой цепочки самопроизвольно собиралась другая такая же. Это не имеет отношения к жизни, но один из примеров "не-молекуляоного" кодирования.
Такие вопросы задавать неприлично. Вероятность существования жизни не на углероде либо 100%, либо 0. Мы этого не можем знать, потому что не от чего отталкиваться. Вокруг нет других примеров жизни. Самим синтезировать "жизнь"? Плиз. Мы еще не знаем, как на нашей планете неживое в живое перешло.
>Мы еще не знаем, как на нашей планете неживое в живое перешло.
Молекулярные компьютеры -> программируемая материя -> абиогенез -> эволюция -> антропогенез -> НТП -> корпоратократия -> теократическая технократия.
> где-то писали, что в принципе возможна жизнь на основе других элементов
даже науку для этого отдельную придумали
Зависит от угла наклона лопастей, если угол между плоскостью лопасти и направлением набегающего потока воздуха будет меньше 45°, то поедет.
>Возможно ли это из-за того, что я болею и надышал случайно хуйней из носа, которая повыкашивала палочки?
Тогда бы выросло что-то другое.
Так-то всё правильно говоришь. Но меня интересует только "природные" альтернативные способы передачи генетического кода потомству.
Влево
Поезд должен двигаться нахуй, пока не сможет в метрическую систему.
С одноразовыми семенами тему слышал, но там вроде как уже первое поколение "потомков" бесплодно.
Искусственно - нереально до нормального, полного, моделирования химических реакций. Эволюционно - нет такой задачи. Впрочем, есть грибы которые меняют пол и вид циклически в несколько поколений.
Почему свинец опасен для человека? Во всех формах? Если, например, не пары вдыхать, а просто с телом соприкасается, то всё равно проблемы будут?
Я хочу утяжелители со свинцом купить, но не уверен, что не отправлюсь.
>но не видно умных вирусов на основе этой технологии?
Не просто умных, вообще вирусов на нейросети и/или способных в эволюцию не видно. Возможно потому что они не нужны, ибо вирусы пишутся под конкретную задачу и уязвимость.
Я не он. Чому лучший то?
Террористы совсем охуели. Стуканул куда положено.
Вот теперь интересно стало, давай, делись инфой. Как вообще происходит заражение? Ведь если кусок свинца разово сожрать, просто высрется ведь, и всё. По моему, это нужно на производстве 10 лет работать, что бы нагрев и мелкая пыль. Еще помню стори, про добавление свинца в топливо, в прошлом веке, на выходе получались легкоусваиваемые соединения, которыми, посредством выхлопа, засрали пол америкашки, растения, водоемы, всю хуйню.
Пилите инфу по теме.
Благотворительность, лол.
Если выпить рюмку ртутии закусить урановым огурцом, она тоже просто высрется. А вот ее соединения приносят тотальный пиздец.
То же и со свинцом - в виде самого металла не опасен, но вот оксиды и всякие соединения растворяются в воде и проникают сквозь кожу, при вдыхании и т.д.
Сразу пиздеца не будет, но если часто тереть свинец о свое потное накачанное тело, будет накопление этого металла у организме.
Если у фотона нет массы покоя и он не может быть остановлен, чем тогда является статическое электромагнитное(электрическое или магнитное) поле?
1280x720, 0:50
С одной стороны холодно, с дугой - жарко. Если заслониться от Солнца(или достаточно далеко улететь) то холодно.
только такие звезды живут очень мало и давно схлопнулись
бамп
Перевожу:
200/4 - 121/6 + 11 = 61 - 121/6 = 41 and 1/6
Где я наебался?
3(2/3) 5()1/2 = 11/3 11/2 == 121/6
ЧО НЕ ТАК БЛЯ
6⋅(1/4)⋅8 - 3⋅(2/3)⋅5(1/2) + 2(2/5)⋅4(7/12) =
(6/4)⋅8 - (6/3)⋅(5/2) + (4/5)⋅(28/12) =
(48/4) - (30/6) + (112/60) =
(48⋅15)/(4⋅15) - (30⋅10)/(6⋅10) + (112/60) =
720/60 - 300/60 + 112/60 =
(720 - 300 + 112)/60 = 532/60 = (133⋅4)/(15⋅4) = (133/15)⋅(4/4) = 1⋅(133/15) =
133/15 = 8⋅(13/15)
Вы видите копию треда, сохраненную 16 марта 2018 года.
Скачать тред: только с превью, с превью и прикрепленными файлами.
Второй вариант может долго скачиваться. Файлы будут только в живых или недавно утонувших тредах. Подробнее
Если вам полезен архив М.Двача, пожертвуйте на оплату сервера.